Anda di halaman 1dari 57

SOAL TRYOUT BOARD PARU

1. Seorang laki-laki 20 tahun datang ke poliklinik dengan keluhan utama sesak napas sejak 1
bulan terakhir. Sesak muncul saat udara dingin dan bila pasien kecapekan. Sesak membaik
bila pasien menggunakan obat semprot. Pasien sudah diketahui menderita asma sejak usia
12 tahun. Pasien tidak rutin kontrol. Sejak satu bulan sesak semakin sering timbul hampir
tiap hari. Sesak saat malam hari 2-3 kali per minggu. Pada pemeriksaan fisik didapatkan
pasien tampak sakit sedang, tekanan darah 110/80 mmHg, nadi 96 x/menit isi cukup, napas
20 x/menit. Pada pemeriksaan paru didapatkan mengi di kedua lapangan paru. Pasien
kemudian mendapatkan inhalasi dengan B2 agonis. Sesudah mendapat inhalasi, keluhan
sesak membaik dan pada pemeriksaan fisik tidak didapatkan mengi.
Tata laksana yang dilakukan selanjutnya adalah:
A. Pasien diberikan terapi rumatan dengan antikolinergik inhaler
B. Pasien diberikan terapi rumatan dengan kortikosteroid inhaler
C. Pasien diberikan terapi rumatan dengan kombinasi kortikosteroid dan B2 agonis kerja
pendek inhaler
D. Pasien diberikan terapi rumatan dengan teofilin serta kombinasi kortikosteroid dan B2
agonis kerja pendek inhaler
E. Pasien diberikan terapi rumatan dengan kortikosteroid oral serta kombinasi
kortikosteroid dan B2 agonis kerja pendek inhaler.

2. Seorang laki-laki berusia 59 tahun dibawa oleh keluarganya ke IGD dalam keadaan demam
tinggi dan menggigil sejak 3 hari yang lalu disertai dengan batuk berdahak warna kuning.
Pasien sudah minum paracetamol tapi demam tidak turun, pasien tidak buang air kecil
dalam 4 jam terakhir. Pada pemeriksaan fisik didapatkan pasien tampak sakit berat; tekanan
darah 100/60mmHg, frekuensi nadi 112x/menit, isi cukup, frekuensi nafas 24x/ menit, suhu
38,5 derajat celcius.
Diagnosis yang paling tepat dari pasien adalah
A. Sepsis
B. Syok sepsis
C. Sepsis berat
D. Infeksi berat
E. Systemic Inflamatory Response Syndroms

3. Seorang perempuan berusia 28 tahun bekerja sebagai buruh pabrik garmen sejak 8 tahun.
Pasien datang dengan keluhan batuk dan sesak disertai napas berbunyi sejak 3 bulan
terakhir. Terdapat batuk kering dan dada terasa tertekan, namun tidak demam. Pasien
sebelumnya telah berobat ke dokter, mendapat terapi obat batuk, antibiotik, dan vitamin,
tetapi keluhan batuk tidak berkurang. Riwayat merokok disangkal, terdapat riwayat
pengobatan TB paru selama 6 bulan, pengobatan teratur dan dinyatakan sembuh. Berat
badan 80 kg, TB 160 cm. PF paru terdapat ekspirasi memanjang dan wheezing pada kedua
lapang paru. Hasil laboratorium klinik dan foto thorax normal. Keluarga tidak ada yang
menderita penyakit alergi. Diagnosis?
A. Asma bronkial
B. SOPT
C. asma akibat kerja
D. pneumonia
E. PPOK
4. Seorang buruh tani, perempuan berusia 56 tahun, datang ke poli dengan keluhan episode
mengi yang sering kambuh setelah bekerja di gudang penyimpanan bahan baku untuk
membuat karung. Sejak setahun yang lalu terdapat batuk kering, namun kadangkala
berdahak berwarna putih. Tak ada demam. Riwayat merokok disangkal. Terdapat riwayat
pengobatan TB paru selama 6 bulan, pengobatan teratur dan telah dinyatakan sembuh. PF
ronki basah halus di basal kedua paru, suara jantung dbn. Lab Hb 12, leukosit 11000,
trombosit 350000, hitung jenis 1/2/2/80/10/5, spirometri FEV1 2,7 liter, FVC 2,4 liter. CXR
corakan bronkovaskular meningkat di kedua hilus paru, sinus kostofrenikus lancip,
diafragma normal. Dx?
A. Bronkitis kronik
B. Bronkiektasis terinfeksi
C. SOPT
D. Pneumonitis hipersensitivitas
E. Asma bronkial eksaserbasi akut

5. Seorang laki laki datang dengan keluhan batuk sesak dengan riwayat merokok lama. PF
didapatkan sesak, respi cepat, PF trakea bergeser ke kanan, suara nafas kanan menurun,
perkusi redup. Dari foto torak didapatkan infiltrate homogen dan radio opak di kanan,
mediastinum dan trakea bergeser ke kanan. Apakah diagnosisnya?
A. efusi pleura kanan
B. tumor paru kanan
C. atelectasis kanan
D. pneumonia lobaris kanan
E. asbestosis

6. Seorang perempuan usia 68 tahun, dikonsulkan ke poliklinik penyakit dalam oleh dokter
umum dengan keluhan batuk dengan sputum produktif dan demam tinggi sejak 2 hari.
Setiap hari pasien menggunakan tiotropium bromida dan albuterol satu kali sehari karena
dikatakan menderita PPOK. Pasien memiliki riwayat diabetes melitus sejak 2 tahun dan saat
ini rutin mengkonsumsi glikuidon 3x30 mg. Pada pemeriksaan fisik didapatkan tekanan
darah 130/80, frekuensi nadi 110x/m, frekuensi napas 33x/m, suhu 40oC. Pemeriksaan paru
sonor, vesikuler, terdapat rhonki basah sedang nyaring di kedua lapangan paru disertai
wheezing. Hasil laboratorium dengan Hb 11.8 g/dL, leukosit 15.000/uL, trombosit
346.000/uL, CRP 30 mg/mL, Saturasi O2 85%
Rontgen thorax penderita
Terapi antibiotik empiris yang tepat pada kasus ini
A Azitromisin intravena
B Ciprofloxacin intravena
C Cefepime dan azitromisin intravena
D Amikasin dan azitromisin intravena
E Aztreonam dan azitromisin intravena

7. Seorang lelaki berusia 54 tahun datang ke IGD dengan keluhan sesak yang disertai nyeri dada
kiri sejak dua jam sebelum masuk rumah sakit. Pada pemeriksaan fisik didapatkan tekanan
darah 130/80 mmHg; frekuensi nadi 110x/menit; frekuensi napas 28x/menit; suhu 36,8oC.
Pemeriksaan paru didapatkan hipersonor pada hemitoraks kiri dengan suara napas vesikuler
melemah sampai hilang. Pasien direncanakan untuk pemeriksaan roentgen torak.
Diagnosis kerja pada pasien yang paling mungkin saat ini adalah:
A. Pleuritis kiri
B. Efusi pleura kiri
C. Pneumotoraks kiri
D. Atelektasis paru kiri
E. Pleuropneumonia kiri

8. Seorang lelaki berusia 35 tahun datang berobat ke poliklinik dengan keluhan sesak napas
sejak seminggu yang lalu. Pasien juga mengeluh batuk disertai dahak kuning kehijauan dan
demam. Pada pemeriksaan fisik paru didapatkan redup di sela iga 4 kanan dan ronkhi di
hemitoraks kanan. Foto toraks menunjukkan infiltrat di bagian atas dan tengah paru kanan
dengan sudut kostofrenikus kanan tumpul. Pasien mengaku dua hari yang lalu sudah
dilakukan pungsi pleura kanan. Hasil laboratorium didapatkan lekosit 14.000/uL, dengan
hitung jenis 0/0/0/85/14/1. Hasil analisis cairan pleura sebagai berikut cairan serous, pH 7,1,
LDH 400; glukosa 40.
Tatalaksana yang tepat pada kondisi pasien saat ini adalah:
A. Antibiotik dan steroid
B. Antibiotik dan pleurodesis
C. Antibiotik dan torakotomi dekortikasi
D. Antibiotik dan pungsi cairan pleura terapeutik
E. Antibiotik dan pemasangan WSD

9. Seorang laki-laki 72 tahun datang ke poliklinik dengan keluhan sesak nafas progresif saat
beraktivitas sejak 3 minggu yang lalu. Pasien batuk produktif, lemah dan anoreksia tapi
menyangkal adanya demam, menggigil dan berkeringat. PF didapatkan tanda vital normal
dan saturasi oksigen normal pada udara ruangan. Tekanan vena jugularis normal dengan
pemeriksaan suara jantung tidak ada kelainan. Trakea bergeser ke kanan dan tidak ada
limfadenopati. Pada pemeriksaan paru kiri tertinggal, fremitus taktil menurun, didapatkan
perkusi redup pada seluruh lapangan paru kiri, suara napas menurun. Pemeriksaan paru-paru
kanan normal. Pada rontgen thoraks, didapatkan gambaran bayangan radio opak pada
seluruh hemithorak kiri dengan pendorongan trakea dan mediastinum ke arah kontralateral.
Pilihan prosedur diagnostik awal yang tepat ada pasien ini adalah:
A. Biopsi transbronkial
B. Biopsi trantorakal
C. CT scan thoraks
D. Torakosentesis
E. Bronkoskopi

10. Seorang laki-laki 50 tahun datang ke poliklinik dengan keluhan batuk berdahak sejak 3
bulan lalu disertai badan meriang dan nafsu makan menurun, berat badan menurun, dan
keringat saat malam hari. Pada PF didapatkan TD 120/80 N 90x/mnt RR 24/mnt, suhu 37,8,
konjunctiva anemis. Perkusi thorax redup dan pada auscultasi didapatkan suara nafas
bronkhial, hasil lab : LED 70 mm/jam; Hb 9; L 10.200; T 405.000; hasil Ro Thorax bercak
bercak infiltrat disertai fibrosis pada daerah segmen apikal lobus kanan atas paru kanan.
Hasil pemeriksaan sputum BTA (-). Terapi yang paling tepat pada kasus ini adalah :
A. OAT dengan regimen 2(HRZE)S/HRZE/5(HR)3E3
B. OAT dengan regimen 2(HRZE)/4(HR)
C. Antibiotik golongan aminoglikosida
D. Antibiotik golongan kuinolon
E. Kortikosteroid

11. Seorang laki-laki 52 tahun datang ke poliklinik datang dengan keluhan batuk, demam
tinggi, disertai sesak nafas. Pada pemeriksaan foto thorax ditemukan adanya perselubungan
homogen dengan air bronkhogram sign di lobus tengah dan bawah kanan. Pasien sebulan
sebelumnya pernah menderita batuk dan sesak nafas. Dan mendapat terapi amoksisilin-asam
klavulanat selama 10 hari dan dexametason. AGD didapatkan hasil pH 7,46 PO2 48 mmHg,
PCO2 46, HCO3 22 saturasi oksigen 90%. Pada terapi empiris, antibiotik yang tepat
adalah :
A. Levofloxacin IV
B. Cefotaxim IV dikombinasikan dengan amikasin
C. Cefpiron IV dikombinasi dengan azithromisin IV
D. Cefepim IV dikombinasi dengan Ciprofloxacin IV
E. Ceftriaxon IV dikombinasi dengan Ciprofloxacin IV

12. Seorang laki-laki 50 tahun dirawat di RS dengan TB paru dan hasil sputum BTA (+). Pada
pemeriksaan didapatkan HIV (+) dengan CD4 85/ul dan belum mendapat terapi ARV.
Tatalaksana awal yang tepat pada pasien adalah
A. 2RHZ/4RH
B. 2RHZE/6HE
C. 2RHZE/4RH
D. 2RHZE/4R3H3
E. 2RHZES/RHZE/5R3H3E3

13. Seorang laki-laki 50 tahun datang ke poliklinik dengan keluhan sesak nafas yang progresif
sejak 3 minggu yang lalu. Pasien batuk produktif, namun menyangkal adanya demam,
menggigil, dan berkeringat. Pada pemeriksaan fisik didapatkan tanda vital dalam batas
normal dan saturasi oksigen dalam udara ruangan. Tekanan vena juguaris normal dan
pemeriksaan jantung tidak ada kelainan. Trakhea bergeser ke kanan dan tidak ada
limfadenopati. Pada pemeriksaan paru kiri tertinggal dan fremitus taktil menurun,
didapatkan perkusi redup pada seluruh lapang paru kiri, suara nafas menurun. Pemeriksaan
paru kanan normal. Pada pemeriksaan rontgen thorax didapatkan gambaran bayangan
radioopak pada seluruh hemithorax kiri dengan pendorongan trakea dan mediastinum ke
arah kontra lateral. Prosedur diagnostik awal yang tepat pada pasien ini adalah :
A. Biopsi transbronkhial
B. Biopsi trans thoracal
C. CT Scan thorax
D. Torakosentesis
E. Bronkhoskopi

14. Seorang laki-laki 60 tahun datan dengan keluhan batuk darah sejak 1 tahun terakhir dan
memberat sejak 1 bulan terakhir. Pasien memiliki riwayat tuberkulosis 5 tahun lalu dan
telah mendapatkan pengobatan di puskesmas selama 6 blan dan dinyatakan sembuh. Sejak 1
tahun terakhir jga menderita kencing manis. Hasil lab didapatkan LED 15 mm/jam, BTA 3x
negatif, hasil pemeriksaan foto thorax lateral didapatkan gambaran air cresent sign.
Diagnosis paling mungkin pada asien ini adalah :
A. Abses paru
B. Mesotelioma
C. Aspergilloma
D. Karsinoma paru
E. Tuberculosis paru

15. Seorang laiki-laki 25 tahun datang ke poliklinik dengan keluhan demam dan batuk yang
hilang timbul disertai penurunan berat badan sejak 2 bulan lalu. Pasien juga mengeluh
kelemahan dikedua tungkai sejak 2 minggu terakhir dan sulit berjalan. Pemeriksaan fisik
didapatkan luka2 pada kulit seperti foto. Pemeriksaan lab didapatkan LED 100 mm/jam.

Kemungkinan diagnosis pasien ini adalah :


A. Abses perinefrik
B. Abses hati piogenik
C. Spondilosis Tuberculosis
D. Infeksi gonococcal diseminata
E. Pseudokista pankreas terinfeksi

16. Seorang laki-laki 41 tahun datang ke IGD dengan keluhan demam tinggi sejak 1 minggu
sebelum masuk rumah sakit. Pasien juga mengeluh batuk dengan dahak berwarna hijau,
badan lemah, nafsu makan turun, dan nyeri dada kanan atas yang hilang timbul. Empat
minggu sebelumnya pasien cabut gigi karena gigi berlubang. Pada pemeriksaan fisik
didapatkan tekanan darah 100/70 mmHg; frekuensi nafas 24x/menit, suhu 39,4 C. Ada
pemeriksaan fisik didapatkan murmur diastolik mitral. Hasil laborat LED 19 mm/jam; Hb
9,8 g/dl; lekosit 21.000/ul. Pemeriksaan thorax PA memperlihatkan gambaran kavitas
multipel yang ukurannya bervariasi pada lobus bawah yang sesuai dengan gambaran emboli
septik paru. Mikrobiologi yang paling sering dari kasus di atas adalah :
A. Basillus medius
B. Streptococcus viridans
C. Haemophylus influenza
D. Staphylococcus aureus
E. Streptococcus pneumonia

17. Seorang perempuan 53 tahun ada efusi pleura dengan hasil analisis cairan efusi : warna
kuning, rivalta (+), sel dominan MN, LDH 380, ADA 85. Diagnosis yang paling mungkin
pada pasien ini adalah :
A. Mesotelioma
B. Parapneumonia
C. Pleuropneumonia
D. Efusi pleura maligna
E. Pleuritis TB

18. Seorang perempuan berusia 30 tahun dibawa ke IGD dengan keluhan sesak napas hebat,
riwayat demam, disertai batuk dahak kuning sejak 4 hari yang lalu. Pasien juga mengeluh
sering sariawan dan diare sejak 1 tahun yang lalu, disertai penurunan berat badan 3 bulan
terakhir. Pada pemeriksaan fisik didapatkan : tekanan darah 90/60 mmHg; frekuensi nadi
180x/menit; frekuensi napas 28x/menit; suhu 38,8°C; berat badan 42 kg; tinggi badan 160
cm. Terdapat oral trush, ronkhi basah kasar di kedua lapangan paru. Hasil laboratorium Hb
10,3 g/dl. Ht 32%;leukosit 1.800/Ul;LED 22 mm/jam; trombosit 373.000/Ul; hitung jenis
0/1/1/86/8/4.
Masalah yang paling mungkin terdapat pada pasien tersebut adalah :
A. TB paru, febrile netropenia
B. Pneumonitis, febrile netropenia
C. Pneumonia, sindrom imunodefisiensi
D. TB paru, sindrom imunodefisiensi
E. Interstitial lung disease, sindrom imunodefisiensi

19. Seorang lelaki usia 50 tahun menyandang DM Tipe 2, datang berobat ke poliklinik dengan
keluhan batuk sejak 4 hari yang lalu disertai sesak napas. PF paru didapatkan
bronkovesikuler pada paru kanan bawah disertai ronki basah nyaring. Pada foto toraks
didapatkan bercak-bercak infiltrat inhomogen pada paru kanan bawah. Mikroorganisme
penyebab infeksi tersering pada kasus pasien tersebut adalah:
A. Bacteroides sp
B. Pneumococcus
C. Enterobactericeae
D. Staphylococcus aureus
E. Mycoplasma pneumoniae
20. Seorang laki-laki datang ke IGD dengan keluhan sesak nafas, batuk darah (+). Riwayat
merokok 15 batang/hari. Pemeriksaan laboratorium Hb 11,3 g/dl; LED 85 mm/jam; lekosit
8.700/ul; trombosit 210.000/ul. Sputum BTA negatif. (dalam soal belum menyebutkan ada
pemeriksaan xfoto) Pemeriksaan penunjang paling tepat yang dilakukan untuk menegakkan
diagnosis awal pada pasien ini adalah :
A. CT scan thorax
B. Bronkhoskopi biopsi
C. Rontgen thorax PA/Lateral
D. Kultur darah dan uji sensitivitas
E. Petanda tumor CEA, NSE, dan Cyfra 21-1

21. Seorang lelaki berusia 67 tahun datang ke IGD dengan keluhan sesak napas memberat sejak
seminggu disertai batuk berdahak warna kuning. Pasien mulai ada keluhan sesak disertai
bunyi "ngik-ngik' sejak 5 tahun yang lalu dan keluhan makin sering timbul sejak 6 bulan ini.
Pasien mempunyai riwayat merokok sejak usia muda dan baru berhenti sejak 5 tahun ini.
Selama ini keluhan sesak mereda dengan menggunakan obat semprot yang diberikan oleh
dokter, namun keluhan seminggu terakhir tidak mereda meski sudah menggunakan obat
semprot tersebut. Pada pemeriksaan fisik didapatkan bentuk dada barrel chest,wheezing
pada seluruh lapangan paru. Hasil AGD: pH 7,32; pO2 65 mmHg; pCO2 60 mmHg; HCO3
30 mEq/L; BE +4; SaO2 96%. Terapi oksigen pada pasien ini adalah :
A. Oksigen 6-8 L/menit dengan simple mask
B. Oksigen 6-8 L/menit dengan rebreathing mask
C. Oksigcn 6-8 L/menit dengan non rebreathing mask
D. Oksigen 1-2 L/menit intermittent dengan nasal kanul
E. Oksigen 4-5 L/menit intermittent dengan nasal kanul

22. Seorang lelaki berusia 45 tahun datang berobat ke poliklinik dengam keluhan batuk-batuk
lama sejak 3 minggu sebelum berobat. Batuk disertai dahak warna kuning kehijauan dan
kadang berbau. Pasien juga mengeluh demam hilang timbul sejak 2 minggu terakhir, dan
selama sakit ini berat badan pasien menurun 5 kg. Pasien merokok 2 bungkus sehari sejak
kurang lebih 20 tahun. Pada pemeriksaan fisis didapatkan konjungtiva pucat, oral hygiene
buruk dengan foetor ex ore (+). Pada pemeriksaan paru didapatkan suara napas bronkial
dengan ronki basah nyaring pada paru kanan bawah. Pemeriksaan laboratorium didapatkan
Hb 10,5 gr/dl; lekosit 19.400/ᶙL dengan dominasi neutrofil; LED 74 mn/jam. Foto toraks
menunjukkan pada paru kanan bawah terdapat kavitas dengan infiltrat di sekitamya disertai
gambaran air-fuid level didalamnya. Kemungkinan diagnosis pada kasus di atas adalah:
A. TB paru
B. Ca paru
C. Abses paru
D. Pneumonia lobaris
E. Bronkiektasis terinfeksi

23. Seorang buruh tani, perempuan berusia 56 tahun, datang berobat ke poliklinik IPD dengan
keluhan episode mengi yang sering kambuh setelah bekerja di gudang penyimpanan bahan
baku untuk membuat karung. Sejak setahun yang lalu terdapat batuk kering, namun
kadangkala berdahak berwarna putih. Tidak terdapat demam. Riwayat merokok disangkal.
Terdapat riwayat pengobatan TBC paru selama 6 bulan teratur dan telah dinyatakan
sembuh. Pemeriksaan auskultasi didapatkan ronki basah halus di basal kedua paru; suara
jantung dalam batas normal. Pemeriksaan penunjang Hb 12 g/dl; leukosit 11.000/pL;
trombosit 350.000/pL; hiturg jenis l/2/2/80/10/5; spirometri FEV1 2,2 liter, FVC 2,4 liter.
Rontgen toraks menunjukkan corakan bronkovaskular meningkat di kedua hilus paru, sinus
kostrofrenicus lancip, diafragma normal. Diagnosis kerja yang paling mungkin pada pasien
ini adalah:
A. Bronkhitis kronik
B. Bronkhiektasis terinfeksi
C. Sindrom obstruksi pasca TB
D. Pneumonitis hipersensitivitas
E. Asma bronkhiale

24. Seorang pria berumur 67 tahun dengan riwayat stroke, tidak bisa berjalan, dan sulit menelan
sekitar 6 bulan, datang dibawa keluarganya berobat ke poliklinik dengan keluhan demam
hilang timbul dalam 3 minggu, disertai nyeri dada dan batuk berdahak warna kuning
kehijauan, kental dan berbau amis, terkadang bercampur darah. Keluarga juga mengeluhkan
kurangnya nafsu makan, sering tersedak saat makan, dan berat badan yang semakin
menurun. Pada pemeriksaan fisik paru ditemukan suara napas bronkial dengan suara
tambahan amforik di lapangan atas paru kanan. Pada pemeriksaan darah tepi, hitung
leukosit 20.200/uL, hitung jenis neutrofil 90 %, limfosit 27%. Diagnosis yang paling
mungkin pada kasus ini adalah:
A. Tuberkulosis paru
B. Penyakit paru obstruktif kronik
C. Bronkiolitis
D. Abses paru
E. Kistik fibrosis

25. Seorang wanita penderita karsinoma payudara, berumur 40 tahun, datang ke unit gawat
darurat dengan keluhan sesak napas, nyeri dada, batuk, dan berkeringat dingin. Keluhan
tersebut baru saja dirasakan secara tiba-tiba saat sedang beristirahat di rumah. Pengukuran
saturasi oksigen dengan pulse oxymetry menunjukkan angka 93%. Pemeriksaan fisik paru
tidak ditemukan kelainan. Elektrokardiografi menunujukkan deviasi aksis ke kanan dan
hambatan berkas kanan sebagian, disertai dengan takikardia. Pemeriksaan lanjutan yang
sebaiknya dilakukan untuk menegakkan diagnosis pada kasus ini adalah, kecuali:
A. Roentgen dada
B. Pemindaian ventilasi-perfusi paru (v/p scan)
C. Analisis gas darah
D. Spirometri
E. Pemeriksaan d-dimer

26. Seorang pria berusia 28 tahun, perokok, datang ke poliklinik penyakit dalam dengan
keluhan nyeri dada dan sesak napas ringan sejak 2 hari yang lalu sepulang dari menonton
konser music rock. Satu orang temannya yang ikut menonton konser tersebut merasakan
keluhan serupa, namun belum mau datang untuk periksa.Tidak ada batuk atau riwayat
penyakit paru-paru sebelumnya. Dari pemeriksaan fisik paru tidak ditemukan kelainan.
Roentgen dada menunjukkan adanya pneumotoraks dengan ukuran < 1 cm 2. Tata laksana
terbaik selanjutnya adalah:
A. Aspirasi sederhana
B. Nebulisasi dengan agonis beta
C. Pemasangan chest tube dengan water seal drianage
D. Torakoskopi
E. Observasi dan ulang roentgen dada dalam 24 jam
27. Seorang pria 72 tahun dengan keluhan sesak napas progresif saat beraktivitas sejak 3
minggu yang lalu. Pasien batuk produktif ringan dan anoreksia tapi menyangkal adanya
demam, menggigil, dan berkeringat. Pada pemeriksaan fisik, dengan tanda-tanda vital
normal dan saturasi oksigen normal pada udara ruangan. Tekanan vena jugularis normal
dengan pemeriksaan suara jantung menjauh tetapi tidak ada kelainan lain. Trakea terletak
pada garis tengah dan tidak ada limfadenopati. Pada pemeriksaan paru kiri tertinggal,
fremitus taktil menurun, didapatkan suara redup pada seluruh lapangan paru kiri, suara
napas menurun. Pemeriksaan paru-paru kanan normal. Setelah X-Ray dada, apakah
tatalaksana awal yang tepat pada pasien ini ?
A. Antibiotik intravena
B. Thoracentesis
C. Bronkoskopi
D. CT Scan dada
E. Bronkodilator

28. Manakah dari pernyataan berikut yang benar mengenai penyebab patogen pneumonia?
A. Pneumonia aspirasi nosokomial : Streptococcus pneumonia
B. Pneumonia pada pusat perawatan kesehatan : bacteroides, fusobacterium
C. Periodontitis berat : Chlamydia pneumoniae, Klebsiella pneumoniae
D. Pneumonia aspirasi nosokomial : Pseudomonas aeruginosa
E. Pneumonia pada pusat perawatan kesehatan : aspergilosis

29. Seorang pria berusia 45 tahun dirujuk ke poli penyakit dalam dari dokter perusahaan. Pasien
sudah 3 tahun bekerja di perusahaan tambang yang sehari-hari menambang di terowongan.
Pasien juga mengeluh demam, batuk berdahak dan kadang disertai sesak nafas. Pasien tidak
merokok namun saat bekerja kadang tidak menggunakan masker dengan alasan tambah
sesak. Pemeriksaan tanda vital didapatkan tekanan darah 120/70 mmHg, denyut nadi 98
kali/menit, regular, frekuensi napas 28 kali/menit, temperatur axilla 37,7 °C. Bacaan foto
toraks didapatkan nodul silikosis di lobus atas kanan paru dengan kalsifikasi. Tes tuberculin
didapatkan hasil (+). Jika pasien ini dipertimbangkan untuk terapi infeksi TB laten, maka
terapi profilaksis yang dianjurkan adalah:
A. Levofloxacin 750 mg/hari
B. INH 300 mg/hari
C. OAT Kategori I
D. Rifampisin 450 mg/hari
E. Belum diberikan sampai hasil BTA +

30. Seorang pria berusia 40 tahun dikonsulkan oleh subdivisi kardiologi dengan kecurigaan
penyakit paru interstisial. Hasil echokardiografi didapatkan hasil hipertensi pulmonal, fraksi
ejeksi ventrikel kiri baik dan tidak didapatkan adanya hipokinetik baik segmental maupun
global. Pemeriksaan tanda vital didapatkan tekanan darah 120/70 mmHg, denyut nadi 100
kali/menit, regular, frekuensi napas 28 kali/menit, temperatur axilla 36,7 °C. Pemeriksaan
fisik jantung didapatkan wide split S2. Hasil rontgen paru tidak didapatkan infiltrat. Apakah
rekomendasi pemeriksaan selanjutnya dan hasil yang diharapkan untuk mendukung
diagnose hipertensi pulmonal karena kemungkinan penyakit interstisial paru:
A. Angiografi paru dengan hasil trombus
B. Analisa Gas Darah dengan hasil alkalosis respiratorik
C. Spirometri dengan hasil restriktif
D. Spirometri dengan hasil obstruktif
E. Scan perfusi paru
31. Seorang pria berusia 63 tahun datang ke instalasi gawat darurat karena sesak nafas yang
memberat. Pasien pernah didiagnosa tumor paru saat periksa di poliklinik paru, namun
menolak semua rencana pemeriksaan lanjutan dan memilih pengobatan alternatif.
Pemeriksaan tanda vital didapatkan tekanan darah 140/90 mmHg, denyut nadi 102
kali/menit, regular, frekuensi napas 34 kali/menit, temperatur axilla 36,3 °C. Pemeriksaan
fisik thorak didapatkan redup di apex kanan atas, belum dilakukan rontgen thorak. Kelainan
daerah kepala leher yang kemungkinan didapatkan pada kasus di atas adalah, kecuali:
A. Ptosis
B. Miosis
C. Enoftalmus
D. Tinitus
E. Anhidrosis hemifasial

32. Laki laki usia 55 tahun datang dengan keluhan sesak napas, sulit menelan, sulit bicara,
anggota tubuh terasa lemah. Dari pemeriksan fisik didapatkan ptosis pada kelopak mata.
Dari hasil laborat didapatkan penurunan gama globulin. Dari hasil ct scan didapatkan massa
berbatas tegas di mediastinum anterior. Diagnosa untuk pasien diatas adalah
A. Timoma
B. Teratodermoid
C. Tiroid
D. Kistamediastinum
E. Neurofibroma

33. Laki usia 65 tahun datang dengan keluhan sering mengantuk di siang hari, sakit kepala di
pagi hari, sering mendengkur dan merasa sering lupa.vital sign. Tensi 160/90 mmHg, nadi
80 x/menit, respirasi 20 x/menit. TB 150. BB 90, LP 100. Terapi untuk kasus diatas adalah
A. CPAP
B. Oksigen nasal kanul
C. Oksigen NRM
D. Beta blocker
E. Calsium chanel blocker

34. Seorang pria berusia 63 tahun datang ke instalasi gawat darurat karena sesak nafas, batuk
dahak warna putih. Riwayat merokok + sudah 30 tahun. Kesadaran somnolen. Tensi 130/90,
Nadi 80 ireguler, RR 30x/menit. Pemeriksaan fisik didapatkan hipersonor, barrel chest.
Dilakukan AGD selama tidur PaO2 78 mmhg dan PO2/FiO2 100. Dokter memberikan
oksigen dari dosis rendah dengan Fio2 0,24, dengan tujuan:
A. Mengurangi efek hipoksia untuk pemicu gerakan bernapas
B. Meningkatkan mismatch ventilasi perfusi
C. Mencegah terjadi retensi CO2
D. Mencegah asidosis respiratorik
E. Semua jawaban diatas benar

35. Seorang paisen lelaki berusia 48 tahun yang dirawat di rumah sakit tiba mengeluh sakit
pada dada sebelah kiri disertai sesak nafas. Paisen pasca operasi tulang panggul dan hanya
tirah baring dalam 6 hari terakhir. Pada pemeriksaan fisik didapatkan tekanan darah 110/70,
frekuensi nadi 116 kali permenit, laju repirasi 26 kali permenit. Pada pemeriksaan paru
tidak didapatkan ronchi. Pasien dilakukan pemeriksaan saturasi oksigen dengan hasil 88 %,
pemeriksaan EKG didapatkan S di lead I, Q dan T inverted di lead III. Hasil foto thorax
dalam batas normal. Gambaran yang sesuai dengan patofisiologi gangguan yang didapatkan
pada pasien ini adalah:
A. Ellis damoseau line
B. Hampton sign
C. Mc Ginn White Pattern
D. Inverted koma sign
E. Horner’s Sign

36. Seorang wanita usia 29 tahun, dengan keluhan batuk dan demam lebih dari 1 bulan, disertai
penurunan berat badan lebih dari 10% dalam 2 bulan terakhir. Saat dilakukan pemeriksaan
labolatorium didapatkan hasil tes HIV positif dengan pemeriksaan ELISA 3 tahap.
Pemeriksaan CD4 menunjukkan angka 250 sel/liter. Dari pemeriksaan darah rutin
didapatkan hasil Hb 9,2, lekosit 8.500, dan angka trombosit 250 ribu. Pemeriksaan LED di
atas normal. Pasien terdiagnosis tuberculosis paru, dan saat ini sedang hamil 34 minggu.
Pilihan terapi pada pasien tersebut adalah :
A. ARV (AZT/3TC/EFV) dan Obat Anti Tuberculosis dimulai bersamaan
B. ARV (AZT/3TC/EFV) diberikan selama 2 mingggu dan diikuti Obat Anti Tuberculosis
C. ARV (TDF/3TC/EFV) diberikan selama 2 mingggu dan diikuti Obat Anti Tuberculosis
D. Obat Anti Tuberculosis diberikan selama 2 mingggu dan diikuti ARV (AZT/3TC/EFV)
E. Obat Anti Tuberculosis diberikan selama 2 mingggu dan diikuti ARV (TDF/3TC/EFV)

37. Seorang pria berusia 60 tahun datang berobat ke poliklinik dengan keluhan batuk sejak 2
tahun yang memberat sejak 3 bulan terakhhir. Batuk dengan dahak berwarna putih kental.
Ketika batuk kadang pasien merasa sesak. Pasien tidak mengeluhkan demam, keringat
malam, atau penurunan berat badan. Pasien merokok 1 bungkus per hari sejak usia 20 tahun
dan baru berhenti 3 bulan yang lalu. Dari pemeriksaan fisik didapatkan tinggi badan 160
cm, berat badan 58 kg, hemodinamik stabil, frekuensi nafas 22x/menit, teratur. Pemeriksaan
paru didapatkan suara nafas vesikuler, tidak ada ronchi, terdapat ekspirasi memanjang. Dari
pemeriksaan spirometri didapatkan :
Spirometri pra bronkodilator Spirometri pasca bronkodilator
FEV1 2.5 Liter FEV1 2. 6 liter
FEV1 Prediction 3.5 liter FEV1 Prediction 3.5 liter
FVC 3.7 liter FVC 3.7 liter
FVC prediction 3.9 liter FVC prediction 3.9 liter

Diagnosis pada pasien ini :


A. Asma
B. Penumonia
C. Penyakit paru restriktitf
D. Infeksi saluran nafas atas
E. Penyakit paru obstruktif kronis

38. Seorang laki-laki, usia 48 tahun datang ke Rumah Sakit dengan keluhan diare, nyeri dan
mukanya memerah. Pada pemeriksaan fisik didapatkan adanya hipertensi, ptosis, miosis
pupil, enoftalmus, anhidrosis yang terjadi ipsilateral serta paraplegia. Pada pemeriksaan
penunjang urin didapatkan adanya Vinil Mandelic Acid (VMA) serta pada foto thorax aspek
lateral didapatkan adanya massa di mediastinum posterior.
Kemungkinan diagnosis pada pasien ini adalah :
A. Germ cell neoplasma
B. Limfoma
C. Kista enterik
D. Ganglioneuroma
E. Neuroblastoma

39. Seorang laki-laki beusia 45 tahun, 3 tahun lalu diketahui terinfeksi HIV dan mendapatkan
terapi antiretroviral. Berat badan menurun 10 kg selama 2 bulan terakhir dan terdapat
penurunan progresif jumlah limfosit CD4 sampai nilai 94/mm2. 1 minggu sebelum berobat
ke IGD, pasien mengeluh sesak nafas, terdapat batuk namun tidak produktif. Pada
pemeriksaan fisik pasien tampak sesak, suhu 38.3, frekuensi nadi 112x/menit, rr 30x/menit,
tekanan darah 130/80. Pada auskultasi didapatkan bising nafas bronkovesikuler dan rhonki
basah halus nyaring pada kedua lapang paru. Pada foto ronthgen thorax didapatkan
gambaran infiltrate interstitial difus luas. Berdasarkan data diatas, kemungkinan penyebab
infeksi oportunistik pada pasien :
A. Sitomegalovirus
B. PCP
C. Mycoplasma pneumonia
D. Cryptococcus
E. Mycobacterium avium complex

40. Seorang lelaki berusia 40 tahun datang berobat ke Poliklinik Penyakit Dalam dengan
keluhan batuk sejak 3 bulan yang memberat sejak 1 bulan terkahir. Batuk terutama muncul
malam hari. Batuk tidak berdahak. Ketika batuk kadang pasien merasa sesak. Tidak ada
demam, keringat malam, atau penurunan berat badan. Data usia 10 tahun kadang muncul
keluhan bersin – bersin dan pilek di pagi hari. Riwayat TBC pada usia 11 tahun dan
mendapat terapi OAT selama 6 bulan. Pasien merokok 4 batang per hari sejak 15 tahun
yang lalu. Ibu pasien memiliki riwayat alergi udang. Pada pemeriksaan fisik didapatkan TB
163 cm; BB 60 kg; hemodinamika stabil; frekuensi napas 18x/menit, teratur, kedalaman
cukup. Pemeriksaan paru didapatkan suara napas vesikuler, tidak ada ronki, terdapat
ekspirasi memanjang. Hasil pemeriksaan spirometri sebagai berikut:

Spirometri pra bronkodilator Spirometri pasca bronkodilator


FEV1 2,4 liter FEV1 2,8 liter
FEV1 prediction 3,1 liter FEV1 prediction 3,1 liter
FVC 3,5 liter FVC 3,6 liter
FVC prediction 3,65 liter FVC prediction 3,65 liter

Diagnosis pada pasien ini:


A. Asma
B. Pneumonia
C. Penyakit paru restriktif
D. Sindrom Obstruksi Pasca-TBC
E. Penyakit paru obtruksi kronik

41. Seorang perempuan berusia 50 tahun dibawa ke IGD karena penurunan kesadaran disertai
sesak memberat dan demam tinggi sejak 1 hari yang lalu. Terdapat keluhan batuk kering
sejak seminggu. Tidak ada riwayat merokok. Tidak ada riwayat penyakit
sebelumnya.pemeriksaan fisik didapatkan kesadaran somnolen, tekanan darah 120/80
mmHg, frekuensi nadi 120x/menit, frekuensi nafas 36x/menit, suhu 39 0C, auskultasi paru
terdapat ronki basah kasar di kedua lapangan paru, tidak ada wheezing. Hasil pemeriksaan
laboratorium menunjukkan leukosit 22.000/ul, analisa gas darah (dengan kanul nasal 0 2 4
liter/menit) : pH 7,2, p02 60 mmHg, pC02 55 mmHg, Hc03 24, 2 mEq/L, saturasi O2 88 %
Prioritas tatalaksana untuk pasien ini adalah
A. Koreksi bikarbonat
B. Pemberian kortikosteroid intravena
C. Diberikan oksigen dengan simple mask 6 liter/menit
D. Intubasi dilanjutkan pemasangan ventilator mekanik
E. Diberikan antibiotic kombinasi cephalosporin generasi 3 dan makrolid

42. Seorang lelaki berusia 35 tahun datang berobat ke Poliklinik Penyakit Dalam dengan
keluhan sesak napas sejak seminggu yang lalu, Pasien juga mengeluh batuk disertai dahak
kuning kehijauan dan demam . Pada pemeriksaan fisik paru didapatkan perkusi redup di
sela iga 4 kanan dan ronkhi di hemi toraks kanan. Hasil laboratorium darah menunjukkan
leukosit 14,000/ mL dengan hitung jenis 0/0/0/85/14/1. LDH serum 250 U/L. Foto toraks
menunjukkan infiltrat kanan dengan perselubungan homogen di paru kanan bawah mulai
sela iga 4, jantung terdorong ke kiri. Pasien mengaku 2 hari yang lalu sudah dilakukan
pungsi pleura kanan keluar cairan kuning sebanyak 400 ml namun pungsi pleura dihentikan
karena pasien batuk. Hasil analisis cairan pleura sebagai berikut : cairan kuning keruh , PH
7,d1 ; LDH 300 U/L; glukosa 60 mg/dL, jumlah sel 2.000 sel /ML dengan dominasi sel
PMN 80 %. Pemeriksaan pewarnaan Gram didapatkan bakteri batang gram positif. Selain
diberikan antibiotik , tatalaksana yang tepat untuk efusi pleura pada pasein saat ini adalah :
A. Pungsi cairan teraupetik ulang
B. Pemasangan water sealed drainage (WSD)
C. Pungsi cairan pleura terapeutik ulang dan dilakukan steroid
D. Pungsi cairan pleura teraupetik ulang dan dilakukan pleurodesis
E. Pungsi cairan pleura teraupetik ulang dan torakotomi dekortikasi

43. Seorang laki-laki 50 tahun datang ke IGD dengan keluhan sesak nafas yang memberat sejak
seminggu. Pasien sudah mengeluhkan sesak nafas 6 bulan terakhir, disertai batuk tanpa
adanya dahak. Pasien merokok 1 bungkus per hari sejak 20 tahun yang lalu. Pasien sudah
beberapa kali berobat ke dokter dan terakhir dikatakan terkena flek paru. Meskipun
pemeriksaan sputum BTA Paru selalu negatif dan telah diobati OAT secara teratur selama 9
bulan, namun keluhan sesak masih dirasakan pasien. Pada pemeriksaan fisik paru
didapatkan suara nafas pokok bronkovesikuler, terdapat ronki basah kasar di kedua basal
paru, tidak ada wheezing. Pada pemeriksaan foto thoraks didapatkan gambaran
bronkiektasis pada kedua basal paru disertai tanda-tanda fibrosis di kedua lapangan paru.
Pemeriksaan penunjang yang paling tepat untuk memastikan diagnosis pasien di atas
adalah:
A. PET CT Scan
B. CT scan toraks tanpa kontras
C. CT scan toraks dengan kontras
D. CT Angiografi arteri pulmonalis
E. EBUS (Endobronkial Ultrasound)

44. Seorang lelaki berusia 55 tahun datang berobat ke poliklinik penyakit dalam dengan
keluhan sesak nafas sejak 2 minggu sebelum berobat. Pasien juga mengeluhkan batuk-batuk
dengan dahak yang kadang-kadang bercampur sedikit darah serta nyeri pada dada kanan.
Pasien merokok 1 bungkus sehari sejak 30 tahun yang lalu. Pemeriksaan fisik paru
didapatkan perkusi paru kanan redup, bising napas menurun, dan pada foto toraks
didapatkan gambaran radio-opak pada hemitoraks kanan disertai trakea dan jantung tertarik
ke arah kanan. Masalah pada pasien ini adalah :
A. Masssa paru kanan
B. Efusi pleura kanan
C. Empiema paru kanan
D. Atelektasis paru kanan
E. Pneumonia lobaris kanan

45. Seorang lelaki berusia 40 tahun, datang berobat ke poliklinik dengan riwayat batuk kronik
sejak 1 bulan sebelum berobat. Batuk dengan dahak yang biasanya berwarna kuning dan
kadang-kadang kehijauan. Tidak ada riwayat mengi, asma, gagal jantung kongestif, dan
penyakit refluks gastroesofageal. Pasien merokok selama 15 tahun sebanyak 1
bungkus/hari. Pada pemeriksaan paru didapati ronki basah kasar nyaring pada basal paru
kanan. Hasil rontgen torak didapatkan gambaran kista kecil-kecil pada paru kanan bawah
disertai bercak-bercak infiltrat disekitarnya. Hasil spirometri: FEV1 80%; FVC 88%.
Berdasarkan data di atas, kemungkinan diagnosis pada pasien adalah:
A. Emfisema paru
B. Asma bronkhiale
C. Bronkhitis kronis
D. Bronkopneumia
E. Bronkiektasis

46. Seorang perempuan berusia 20 tahun datang ke IGD dengan keluhan sesak nafas memberat
sejak satu hari terakhir. pasien lebih nyaman duduk di bandingkan berbaring. Lima hari
yang lalu pasien mulai mengeluh batuk, berdahak kuning, pasien diketahui menderita
asama. Pada pemeriksaan fisik di dapat kesadaran kompos mentis, tampak sakit sedang,
tekanan darah 110/70mmHg. Frekuensi nadi 110x/menit. Irama reguler, frekuensi nafas
24x/menit reguler. Saturasi oksigen perifer 94%, auskultasi paru vesikuler, terdapat ronki
basah kasar di paru kanan, terdapat wheezing pada kedua paru. Selain diberikan
suplementasi oksigen dengan target saturasi 93-95%, tatalaksana awal yang paling tepat
diberikan pada pasien ini adalah:
A. Inhalasi agonasi ß2 kerja pendek dan inhalasi kortikosteroid yang di ulang setiap 20
menit selama satu jam
B. Inhalasi agonasi ß2 kerja pendek yang di ulang setiap 20 menit selama satu jam dan
pemberian aminofilin intravena
C. Inhalasi agonasi ß2 kerja panjang yang di ulang setiap 20 menit selama satu jam dan
kortikosteroid sistemik setara prednison 1mg per kg BB
D. Inhalasi agonasi ß2 kerja pendek di ulang setiap 20 menit selama satu jam dan
kortikosteroid sistemik secara prednison 1mg per kgBB
E. Inhalasi agonasi ß2 kerja panjang yang di ulang setiap 20 menit selama satu jam.
Kortikosteroid sistemik setara prednison 1mg per kg BB. Dan pemberian aminofilin
intravena.

47. Seorang perempuan berusia 48 tahun pasca operasi torakotomi 1 hari yang lalu, dalam
evaluasi didapatkan cairan bewarna putih susu dari selang WSD. Pada pemeriksaan fisik
didapatkan pasien dalam ventilator; frekuensi napas 22x/menit; suhu 36,5 oC; tekanan darah
110/70 mmHg; frekuensi nadi 96x/menit; perkusi paru redup pada sela iga 4 paru kiri;
auskultasi paru vesikuler melemah di sela iga 4 paru kiri. Foto toraks menunjukkan
perselubungan homogen di basal paru kiri, jantung terdorong ke kanan. Pemeriksaan cairan
pleura yang paling tepat untuk memastikan diagnosis pada pasien adalah:
A. Konsentrasi kolestrol
B. Konsentrasi trigliserida
C. Adenosine deaminase (ADA)
D. Pemeriksaan kultur cairan pleura
E. Pemeriksaan Gram dan Ziehl- Nielsen

48. Seorang perempuan berusia 50 tahun dibawa ke IGD karena penurunan kesadaran disertai
sesak memberat dan demam tinggi sejak 1 hari yang lalu. Terdapat keluhan batuk kering
sejak seminggu. Tidak ada riwayat merokok. Tidak ada riwayat penyakit
sebelumnya.pemeriksaan fisik didapatkan kesadaran somnolen, tekanan darah 120/80
mmHg, frekuensi nadi 120x/menit, frekuensi nafas 36x/menit, suhu 39 0C, auskultasi paru
terdapat ronki basah kasar di kedua lapangan paru, tidak ada wheezing. Hasil pemeriksaan
laboratorium menunjukkan leukosit 22.000/ul, analisa gas darah (dengan kanul nasal 0 2 4
liter/menit) : pH 7,2, p02 60 mmHg, pC02 55 mmHg, Hc03 24, 2 mEq/L, saturasi O2 88 %.
Prioritas tatalaksana untuk pasien ini adalah
A. Koreksi bikarbonat
B. Pemberian kortikosteroid intravena
C. Diberikan oksigen dengan simple mask 6 liter/menit
D. Intubasi dilanjutkan pemasangan ventilator mekanik
E. Diberikan antibiotic kombinasi cephalosporin generasi 3 dan makrolid

49. Seorang pasien lelaki berusia 41 tahun dikonsulkan ke Sejawat Bedah dengan keterangan
dijumpai nodul paru soliter dari hasil CT-Scan abdomen pasien. Pasien berobat ke sejawat
Bedah karena sering nyeri perut berulang. Pasien tidak mempunyai keluhan batuk atau
sesak, namun mempunyai kebiasaan merokok 1 bungkus sehari sejak usia 20 tahun, yang
baru dihentikan sejak 3 hari yang lalu. Pasien kemudian dilakukan CT-Scan toraks dengan
hasil nodul paru soliter non kalsifikasi paru kanan bawah yang tidak menyangat dengan
pemeriksaan kontras, diameter 7 mm, tepi tajam. Keputusan klinis yang tepat untuk
dilakukan pada pasien adalah
A. Melakukan bronkoskopi
B. Meminta pemeriksaan PET-scan
C. Melakukan biopsi transtorakal
D. Merujuk ke dokter bedah toraks untuk reseksi nodul paru
E. Observasi dan meminta pasien mengulang CT-scan toraks 3-6 bulan lagi

50. Seorang lelaki berusia 35 tahun datang berobat ke Poliklinik Penyakit Dalam dengan
keluhan sesak napas sejak seminggu yang lalu, Pasien juga mengeluh batuk disertai dahak
kuning kehijauan dan demam . Pada pemeriksaan fisik paru didapatkan perkusi redup di
sela ga 4 kanan dan ronkhi di hemitoraks kanan. Hasil laboratorium darah menunjukkan
leukosit 14,000/ mL dengan hitung jenis 0/0/0/85/14/1. LDH serum 250 U/L. Foto toraks
menunjukkan infiltrat kanan dengan perselubungan homogen di paru kanan bawah mulai
sela iga 4, jantung terdorong ke kiri. Pasien mengaku 2 hari yang lalu sudah dilakukan
pungsi pleura kanan keluar cairan kuning sebanyak 400 ml namun pungsi pleura dihentikan
karena pasien batuk. Hasil analisis cairan pleura sebagai berikut : cairan kuning keruh , PH
7,d1 ; LDH 300 U/L; glukosa 60 mg/dL, jumlah sel 2.000 sel /ML dengan dominasi sel
PMN 80 %. Pemeriksaan pewarnaan Gram didapatkan bakteri batang gram positif. Selain
diberikan antibiotik, tatalaksana yang tepat untuk efusi pleura pada pasein saat ini adalah :
A. Pungsi cairan teraupetik ulang
B. Pemasangan water sealed drainage (WSD)
C. Pungsi cairan pleura terapeutik ulang dan dilakukan steroid
D. Pungsi cairan pleura teraupetik ulang dan dilakukan pleurodesis
E. Pungsi cairan pleura teraupetik ulang dan torakotomi dekortikasi

51. Seorang laki-laki 50 tahun datang ke IGD dengan keluhan sesak nafas yang memberat sejak
seminggu. Pasien sudah mengeluhkan sesak nafas 6 bulan terakhir, disertai batuk tanpa
adanya dahak. Pasien merokok 1 bungkus per hari sejak 20 tahun yang lalu. Pasien sudah
beberapa kali berobat ke dokter dan terakhir dikatakan terkena flek paru. Meskipun
pemeriksaan sputum BTA Paru selalu negatif dan telah diobati OAT secara teratur selama 9
bulan, namun keluhan sesak masih dirasakan pasien. Pada pemeriksaan fisik paru
didapatkan suara nafas pokok bronkovesikuler, terdapat ronki basah kasar di kedua basal
paru, tidak ada wheezing. Pada pemeriksaan foto thoraks didapatkan gambaran
bronkiektasis pada kedua basal paru disertai tanda-tanda fibrosis di kedua lapangan paru.
Pemeriksaan penunjang yang paling tepat untuk memastikan diagnosis pasien di atas
adalah:
A. PET CT Scan
B. CT scan toraks tanpa kontras
C. CT scan toraks dengan kontras
D. CT Angiografi arteri pulmonalis
E. EBUS (Endobronkial Ultrasound)

52. Seorang lelaki berusia 55 tahun datang berobat ke poliklinik penyakit dalam dengan
keluhan sesak nafas sejak 2 minggu sebelum berobat. Pasien juga mengeluhkan batuk-batuk
dengan dahak yang kadang-kadang bercampur sedikit darah serta nyeri pada dada kanan.
Pasien merokok 1 bungkus sehari sejak 30 tahun yang lalu. Pemeriksaan fisik paru
didapatkan perkusi paru kanan redup, bising napas menurun, dan pada foto toraks
didapatkan gambaran radio-opak pada hemitoraks kanan disertai trakea dan jantung tertarik
ke arah kanan. Masalah pada pasien ini adalah :
A. Masssa paru kanan
B. Efusi pleura kanan
C. Empiema paru kanan
D. Atelektasis paru kanan
E. Pneumonia lobaris kanan

53. Seorang lelaki berusi 65 tahun berobat ke Poliklinik Penyakit dalam karena batuk dan sesak
nafas yang makin memberat sejak 2 bulan terakhir. Berat badan pasein menurun 10 kg dan
sudah dua kali keluar darah merah segar saat batuk. Pasien seorang perokok berat ( 2
bungkus /hari ). Pada foto thoraks terdapat massa di lobus atas kiri dengan pelebaran
mediastinum dan efusi pleura kiri setinggi sela iga 4. Pada pemeriksaan Trans Thoracal
Needle Aspiration (TTNA) dan sitologi cairan pleura didapatkan hasil adenilarsinoma.
Pemeriksaan Epidermal Growth Factor Receptor (EGFR) dan status Anaplastic
Lymphomma Kinase (ALK) negatif, Hasil bone scan negatif. Pilihan tatalaksana yang tepat
untuk pasien adalah :
A. Radiasi
B. Torakotomi
C. Kemoterapi
D. Kemoradiasi
E. Terapi target
54. Seorang perempuan berusia 23 tahun G1P0A0 hamil 22 mingggu datang berobat ke
poliklinik penyakit dalam karena sesak nafas dan batuk kering sejak seminggu yang lalu.
Pasien memiliki riwayat asma sejak usia 11 tahun, sehingga rutin menggunakan inhaler
kortikosteroid dan LABA sejak awal kehamilan. Pasien menghentikan pengobatan inhaler
sendiri karena mengkhawatirkan efek samping obat tersebut pada janin. Pada pemeriksaan
fisik didapatkan frekuensi nafas 22 x/mnt, ronki tidak ada, terdapat wheezing dikedua paru,
pada pemeriksaan spirometri didapatkan FVC prediction 90%, FEV1 prediction 50%,
FEV1/FVC 55%, sesudah diterapi dengan inhalasi salbutamol, FEV1 prediction 90%
sedangkan FVC prediction tetap 90%. Setelah asma akut teratasi, terapi rumatan yang tepat
diberikan semasa kehamilan adalah:
A. Terapi kortikosteroid inhaler
B. B2 agonis kerja pendek nhaler
C. Hentikan semua pengobatan asma
D. Kombinasi B2 agonis kerja panjang dengan steroid inhaler
E. Kombinasi ipratropium bromide dan B2 agonis kerja panjang inhaler

55. Seorang pria berusia 58 tahun dibawa oleh keluarganya ke UGD dengan keluhan utama
sesak napas sejak 10 hari sebelum masuk rumah sakit. Pasien juga mengeluh batuk-batuk
dengan dahak bercampur darah disertai nyeri pada dada kanan. Pasien merokok 15 batang
per hari sejak 25 tahun yang lalu. Pemeriksaan tanda vital didapatkan tekanan darah 100/60
mmHg, denyut nadi 110 kali/menit, regular, frekuensi napas 28 kali/menit, temperatur axilla
37,7 °C. Pemeriksaan paru didapatkan perkusi paru kanan redup, suara napas menurun, dan
pada foto toraks didapatkan gambaran radioopak pada hemitoraks kanan disertai pergeseran
trakea dan jantung ke arah kiri. Hasil CT-Scan thoraks dicurigai keganasan pada paru. Bila
kondisi pasien seperti ini, keganasan parunya mencapai stadium?
A. IIA
B. IIB
C. IIIA
D. IIIB
E. IV

56. Seorang perempuan berusia 42 tahun, dengan TB 156 cm, BB 76 kg, datang ke IGD dengan
keluhan sesak napas dan nyeri dada yang memberat setiap kali pasien menarik nafas sejak 1
jam SMRS. Pasien pengguna kontrasepsi hormonal. Pada pemeriksaan fisik didapatkan fisik
di dapatkan edema pada tungkai kiri (paha ke bawah). Gambaran EKG dan foto thoraks
sebagai berikut :
Diagnosis yang paling mungkin pada pasien ini adalah :
A. Infark miokard lama
B. Diseksi aorta
C. Perikarditis akut
D. Post infark angina
E. Emboli pulmonal

57. Laki-laki 51 tahun datang ke poliklinik dengan keluhan sesak nafas yang memberat sejak 7
hari lalu disertai demam dan batuk berdahak. Keluhan sesak nafas sudah dialami sejak usia
20 tahun terutama bila terpapar debu dan membaik bila menggunakan salbutamol semprot.
Sejak 1 tahun terakhir keluhan sesak nafas dirasakan muncul setiap hari. Dan tidak
membaik meskipun menggunakan salbutamol inhaler. Pada pemeriksaan paru didapatkan
sela iga melebar, suara nafas bronkhial, disertai wheezing. Pemeriksaan FEV1 2,3L/menit
dan post bronkhodilator FEV1 2,4 L/mnt. Dari hasil kultur dahak didapatkan Aspergillus.
Pemeriksaan lanjutan apa yang anda sarankan:
A. IgE serum
B. Eosinofil darah
C. Spirometri serial
D. Bronkoskopi
E. Foto thoraks

58. Seorang lelaki usia 70 tahun selama ini tinggal di panti jompo, datang berobat ke poliklinik
dengan keluhan batuk sejak 4 hari yang lalu disertai sesak napas. PF paru didapatkan
bronkovesikuler pada paru kanan bawah disertai ronki basah nyaring. Pada foto toraks
didapatkan bercak-bercak infiltrat pada paru kanan bawah. Mikroorganisme penyebab
infeksi tersering pada kasus pasien tersebut adalah:
A. Staphylococcus aureus
B. Streptococcus pneumonia
C. Mycoplasma pneumonia
D. Bacteroides fragilis
E. Pseudomonas aeruginosa

59. Seorang laki-laki usia 70 tahun, dirawat di RS karena stroke iskemik akut selama 14 hari.
Pada hari ke 14 perawatan, pasien mengalami demam dan dari pemeriksaan fisik didapatkan
ronki basah kasar di lapangan paru kanan. Hasil foto thoraks terbaru dibandingkan foto
thoraks saat hari pertama MRS, didapatkan pertambahan infiltrat pada paru kanan.
Mikroorganisme tersering sebagai penyebab pada kasus seperti ini adalah:
A. Klebsiella pneumonia
B. Haemophilus influenza
C. Legionella pneumophilla
D. Mycoplasma pneumonia
E. Staphylococcus aureus

60. Seorang perempuan usia 53 tahun datang berobat ke poliklinik dengan keluhan sesak napas
yang semakin hebat sejak 3 hari SMRS. Pasien mengeluh sesak napas yang tidak berkurang
dengan istirahat 3 minggu terakhir, batuk berdahak putih kental, tidak disertai mengi, dan
pasien lebih nyaman berbaring ke sebelah kanan. Pasien kadang demam, nafsu makan dan
berat badan semakin turun. Riwayat makan OAT dan kontak dengan penderita TB
disangkal. Dari pemeriksaan hasil analisa cairan pleura didapatkan cairan warna kuning,
hitung sel MN 95%, glukosa 20 mg/dl, Rivalta (+), LDH 380 u/l, ADA 85. Tatalaksana
yang paling tepat adalah:
A. Antibiotik
B. NSAID
C. OAT
D. OAT dan steroid
E. Kemoterapi

61. Seorang wanita, 23 tahun, bekerja sebagai perawat RS yang bekerja di ruang perawatan TB,
menjalani pemeriksaan skrining TB karena riwayat kontak dengan penderita TB aktif. Satu
tahun yang lalu hasil mantoux tes indurasi 3 mm. Saat ini hasil mantoux tes didapatkan
indurasi 18 mm dalam 48 jam. Saat ini tidak ada gejala khusus yang dialami wanita ini.
Tidak memiliki riwayat pengobatan TB sebelumnya. Tata laksana selanjutnya adalah:
A. Foto thoraks
B. Ulangi tes Mantoux 2 bulan kemudian
C. INH 300 mg selama 6 bulan
D. Foto thoraks dan INH 300 mg selama 3 bulan
E. Diberikan antibiotik empiris

62. Seorang perempuan 29 tahun datang ke poliklinik karena batuk dan sesak nafas yang
semakin memberat sejak 2 bulan terakhir. Pasien juga mengeluh sakit di punggung yang
hilang timbul. Berat badan pasien menurun 10 Kg dan sudah 2 kali keluar darah segar saat
batuk. Tidak ada riwayat merokok. Pada foto torak terdapat massa di lobus atas kiri dengan
pelebaran mediastinum dan efusi pleura kiri setinggi sela iga 4. Pada pemeriksaan trans
thoracal needle aspiration (TTNA) dan sitologi cairan pleura didapatkan hasil small-cell
Ca. Dan didapatkan pula multipel nodul di hasil CT-Scan otak dan gambaran kompresi
vertebral pada foto thorakolumbal. Tatalaksana yang tepat pada pasien ini adalah:
A. Radiasi
B. Kemoradiasi
C. Kemoterapi
D. Operasi disertai kemoterapi adjuvan
E. Suportif saja

63. Seorang perempuan 18 tahun mengeluh sesak napas berbunyi. Dalam 1 bulan terakhir
serangan mengi sering berulang 2-3 x/minggu. Sering terbangun malam hari karena batuk-
batuk dan tidak enak di dada. Keluarga dengan riwayat atopik (+). Penderita selama ini
menggunakan reliever berupa Salbutamol inhaler untuk meringankan keluhan
2-3-x/minggu. Tatalaksana selanjutnya yang tepat adalah:
A. Tambah inhalasi budesonide dosis rendah
B. Tambah kombinasi inhalasi formoterol dan budesonide dosis sedang
C. Tambah inhalasi ipratropium bromide
D. Tambah metilprednisolon oral dosis rendah
E. Tambah anti IgE

64. Seorang laki-laki, 72 tahun masuk ke RS dengan keluhan sesak nafas, keluhan sesak sudah
dialami sejak 6 bulan lalu, dan memberat 1 minggu terakhir, batuk berdahak sejak sejak
lama, dan 1 minggu terakhir dahak bertambah banyak dengan warna kehijauan, Riwayat
merokok sejak muda + 1 bungkus perhari dan telah berhenti 1 tahun lalu. Pemeriksaan fisis
paru Inspeksi: Simetris, sela iga melebar dan mendatar, Palpasi: vocal fremitus sama kiri &
kanan, Perkusi: hipersonor, Auskultasi: suara nafas bronkial, dengan wheezing difus kedua
paru. Tatalaksana yang tepat pada pasien ini adalah:
A. Injeksi steroid, SABA, SAMA
B. Antibiotik, injeksi steroid, mukolitik
C. Antibiotik, injeksi steroid, SABA, SAMA, mukolitik
D. Antibiotik, steroid inhalasi, SABA, SAMA, mukolitik
E. Antibiotik, steroid inhalasi, SABA, SAMA, mukolitik, aminofilin iv

65. Seorang laki-laki 60 tahun datang ke IGD dengan keluhan sesak hebat sejak 1 hari. Selain
itu pasien juga mengeluh batuk dengan dahak putih sejak 3 tahun dan memberat 3 hari
dengan jumlah dahak bertambah banyak disertai perubahan warna menjadi kehijauan.
Pasien memiliki riwayat sesak nafas bila beraktivitas berat. Pada pemeriksaan fisik
didapatkan kesadaran kompos mentis, tekanan darah 140/80 mmHg, frekuensi nafas 32
x/mnt, frekuensi nadi 101 x/mnt, suhu aksiler 38,1 oC. Pemeriksaan paru tampak retraksi
sela iga dan wheezing. Selain itu juga didapatkan jari tabuh. Hasil analisis gas darah pH
7,31; pO2 62 mmHg; pCO2 58 mmHg; HCO3 19,2 mmol/L; BE +1. Pada pemeriksaan foto
thorak didapatkan gambaran tampak sela iga melebar, diafragma datar, parenkim paru
hiperradiolusen. Tatalaksana awal selain pemberian oksigen pada pasien in adalah :
A. Antibiotik dan drip aminofilin
B. Injeksi steroid dan drip aminofilin
C. Drip aminofilin dan inhalasi formoterol-budesonide
D. Inhalasi formoterol-budesonide dan SAMA
E. Inhalasi SAMA dan mukolitik

66. Seorang pria 24 tahun datang ke ruang gawat darurat mengeluh sesak napas dan nyeri sisi
kanan dada. Gejala mulai tiba-tiba sekitar 2 jam sebelumnya. Nyeri ini lebih memberat saat
inspirasi. Demam atau menggigil disangkal dan kaki bengkak tidak ada. Dia tidak memiliki
masa lalu riwayat kesehatan tetapi merokok 1 bungkus rokok setiap hari. Pada pemeriksaan
fisik, takipnoe dengan pernapasan yang cepat 24 x / menit. saturasi oksigen nya adalah 94%
pada udara ruangan. suara nafas menurun di paru-paru kanan, dan ada hypersonor pada
perkusi. Diagnosis yang paling mungkin pada pasien ini adalah:
A. Massa paru kanan
B. Efusi pleura kanan
C. Empiema kanan
D. Pneumothoraks kanan
E. Fluidopneumothoraks kanan
67. Seorang pria berusia 58 tahun dibawa oleh keluarganya ke UGD dengan keluhan utama
sesak napas sejak 10 hari sebelum masuk rumah sakit. Pasien juga mengeluh batuk-batuk
dengan dahak bercampur darah disertai nyeri pada dada kanan. Pasien merokok 15 batang
per hari sejak 25 tahun yang lalu. Pemeriksaan tanda vital didapatkan tekanan darah 100/60
mmHg, denyut nadi 110 kali/menit, regular, frekuensi napas 28 kali/menit, temperatur axilla
37,7 °C. Pemeriksaan paru didapatkan perkusi paru kanan redup dan membentuk gambaran
garis Ellis Dameisoux, suara napas menurun, dan pada foto toraks didapatkan gambaran
radioopak pada hemitoraks kanan disertai pergeseran trakea dan jantung ke arah kiri.
Diagnosis yang paling mungkin adalah:
A. Efusi pleura kanan
B. Massa paru kanan
C. Atelektasis paru kanan
D. Empiema kanan
E. Pleuropneumonia kanan

68. Seorang laki-laki 55 tahun datang ke poliklinik dengan keluhan sesak nafas sejak 2 minggu
sebelum berobat. Pasien juga mengeluh batuk-batuk dengan dahak yang kadang-kadang
bercampur dengan sedikit darah serta nyeri pada dada kanan. Pasien merokok 1 bungkus per
hari sejak 30 tahun yll. Pasien juga mengalami demam sumer-sumer disertai penurunan
nafsu makan dan penurunan BB drastis. Pemeriksaan dada tampak asimetris. Pada inspeksi
hemitorak kanan tertinggal. Pemeriksaan fisik paru didapatkan perkusi paru kanan redup,
bising nafas menurun, dan pada foto toraks didapatkan gambaran radio-opak pada gambaran
hemitoraks kanan disertai pergeseran trakea dan jantung ke arah kiri. Diagnosis yang paling
mungkin:
A. Pneumothoraks kanan
B. Pneumonia kanan
C. Massa paru kanan
D. Kolaps paru kanan
E. Schwarte paru kanan

69. Seorang pria 24 tahun datang ke ruang gawat darurat mengeluh sesak napas dan nyeri sisi
kanan dada. Gejala mulai tiba-tiba sekitar 2 jam sebelumnya. Nyeri ini lebih memberat saat
inspirasi. Demam atau menggigil disangkal dan kaki bengkak tidak ada. Dia tidak memiliki
masa lalu riwayat kesehatan tetapi merokok 1 bungkus rokok setiap hari. Pada pemeriksaan
fisik, takipnoe dengan pernapasan yang cepat 24 x / menit. Saturasi oksigen nya adalah 94%
pada udara ruangan. suara nafas menurun di paru-paru kanan, dan ada hypersonor pada
perkusi. Patofisiologi yang mungkin mendasari terjadinya kondisi ini adalah:
A. Keradangan pada pleura
B. Pecahnya bleb
C. Obstruksi saluran napas kecil
D. Obstruksi saluran napas besar
E. Gangguan ventilasi karena rangsangan sentral

70. Seorang laki-laki 41 tahun datang ke UGD dengan keluhan demam tinggi sejak 1 minggu
SMRS. Penderita juga mengeluh adanya batuk-batuk dengan dahak berwarna hijau dan
berbau, badan terasa lemah, nafsu makan menurun. Nyeri dada kanan atas dirasakan hilang
timbul. Empat minggu sebelumnya penderita cabut gigi karena gigi berlubang. Pada
pemeriksaan fisik ditemukan TD 100/70 mmHg, T: 39,4 C, pernafasan 24 kali/menit.
Laboratorium menunjukkan Hb : 9,8 mg/dl, leukosit 21.000, LED: 19 mm/jam.
Pemeriksaan Ro thoraks PA memperlihatkan cavitas irregular dengan gambaran air fluid
level. Etiologi paling sering dari kasus di atas adalah:
A. Staphylococcus aureus
B. Haemophilus influenza
C. Streptococcus pneumonia
D. Streptococcus viridans
E. Bacteroides fragilis

71. Pasien laki-laki 52 tahun datang dengan membawa rujukan dari dokter neurologi dengan
massa mediastinum. Pasien sebelumnya mengeluh sulit menelan dan kelopak mata sulit
dibuka sejak 3 bulan. Pasien mengeluh sesak nafas dan batuk kering, dengan riwayat
tuberkulosis dan telah selesai menjalani pengobatan tuberkulosis sejak 3 bulan lalu. Pada
pemeriksaan didapatkan perkusi pekak pada linea parasternal kiri atas. Rontgen thorax PA
dan lateral tampak bayangan radioopak pada mediastinum superior anterior dengan batas
tegas dan disertai deviasi trakhea ke kanan. Diagnosis paling mungkin pada pasien ini
adalah:
A. Limfoma
B. Teratoma
C. Timoma
D. Abses paru
E. Tuberculoma

72. Seorang laki-laki 55 tahun masuk ke ICU setelah 3 hari MRS di bangsal rawat inap dengan
demam sejak 2 hari, sesak napas dan batuk yang memberat. Selama 3 hari perawatan di
bangsal, pasien telah mendapatkan oksigen dan antibiotik empirik. Di ICU dilakukan
intubasi endotracheal, diberikan resusitasi cairan dan mendapat obat vasopressor, dan
pernapasan mekanik. Pada pemeriksaan ditemukan suhu 40,2, TD 90/60, N 102x/m, RR
36x/m, SpO2 95% dengan bantuan ventilasi dan fraksi inspirasi oksigen 50 %. Pada kedua
paru terdengar ronkhi, clubbing dan sianosis (-). Hasil foto thoraks di ICU didapatkan
infiltrat difus pada kedua lapang paru. PaO2/FiO2 <200. Kondisi apakah yang dialami
pasien ini:
A. Acute Lung Injury
B. Acute Respiratory Distress Syndrome
C. Tranfusion-related Lung Injury
D. Pleuropenumonia
E. Gagal napas

73. Seorang perempuan 28 tahun bekerja sebagai buruh pabrik garmen sejak 8 tahun, pasien
datang berobat ke poli IPD dengan keluhan batuk dan sesak disertai napas berbunyi yang
hilang timbul sejak 3 bulan terakhir. Terdapat batuk kering namun tidak ada demam. Pasien
sebelumnya telah berobat ke dokter mendapat terapi obat batuk antibiotik dan vitamin tetapi
keluhan batuk tidak berkurang. Riwayat merokok disangkal. Terdapat riwayat pengobatan
TBC paru 6 bulan teratur dan dinyatakan sembuh 5 tahun yang lalu. Keluarga tidak ada
menderita penyakit alergi. BB 65 kg. TB 160 cm. Pemeriksaan fisik paru terdapat ekspirasi
memanjang dan wheezing pada kedua lapang paru. Hasil laboratorium kimia klinik dan
rontgen thorax dalam batas normal. Pemeriksaan lanjutan apa yang dianjurkan untuk
mendiagnosis pasien ini:
A. Skin prick test
B. Patch test
C. IgE total serum
D. Spirometri
E. Bronkoskopi

74. Laki-laki 51 tahun datang ke poliklinik dengan keluhan sesak nafas yang memberat sejak 2
hari lalu. Keluhan sesak nafas sudah dialami sejak usia 20 tahun terutama bila terpapar debu
dan membaik bila menggunakan salbutamol. Sejak 1 tahun terakhir keluhan sesak nafas
dirasakan muncul setiap hari. Dan tidak membaik meskipun menggunakan salbutamol
inhaler. Pada pemeriksaan paru didapatkan sela iga melebar, suara nafas bronkhial, disertai
wheezing. Pemeriksaan FEV1 2,3L/menit dan post bronkhodilator FEV1 2,4 L/mnt.
Patofisiologi yang mendasari kondisi ini adalah:
A. Obstruksi saluran napas kecil-sedang
B. Peningkatan resistensi saluran napas
C. Sebukan sel-sel radang yang menghasilkan sitokin inflamasi
D. Inflamasi karena degranulasi sel mast dan basofil
E. Kerusakan parenkim paru akibat inflamasi

75. Seorang laki-laki 32 tahun ke IGD dengan keluhan sesak napas memberat tiba-tiba dan
nyeri dada saat batuk. Sesak napas saat aktivitas 3 bulan, batuk tidak produktif, anoreksia
dan penurunan berat badan 7 kg dalam 3 bln. Merokok 1-2 bungkus perhari, kadang minum
alkohol, faktor risiko HIV tdk ada. CxR: pneumothorax kanan 80%, infiltrat nodular di kiri
basal. Setelah pemasangan WSD, CT scan: infiltrat nodular bilateral di basal paru dan
multipel kista kecil di apex. Pemeriksaan penunjang apa yang paling tepat untuk
mendiagnosis pasien ini:
A. Uji klorida keringat
B. Kadar alpha-1 antitrypsin
C. CT-Scan thoraks resolusi tinggi
D. Kultur dahak
E. FNAB pada nodul paru
76. Temuan apakah yang anda harapkan ditemukan pada pemeriksaan bronchoalveolar lavage
pada pasien perdarahan alveolar diffuse?
A. Pneumosit atipikal hiperplastik tipe II
B. Ferruginous bodies
C. Makrofag yang mengandung hemosiderin
D. Limfositosis dnegan peningkatan rasio CD4:CD8
E. Gambaran seperti susu dengan foamy macrophage

77. Laki-laki berusia 42 tahun datang dengan dispnea progresif, demam sumer-sumer, dan
penurunan berat badan selama lebih dari 6 bulan. Pasien juga mengeluhkan adanya batuk
kering, namun terkadang batuk juga disertai dengan sputum mukoid. Tidak ada riwayat
penyakit sebelumnya. Pasien tidak merokok. Pada pemeriksaan fisik ditemukan pasien
dispnea dengan usaha napas minimal. Suhu tubuh pasien: 37,9°C, saturasi oksigen: 91%
pada udara ruangan. Rhonki halus (+). Pemeriksaan laboratorium ditemukan
hypergammaglobulinemia poliklonal dan hematokrit 52%. CT scan thorax menunjukkan
adanya infiltrat alveolar bilateral terutama daerah perihilus dengan gambaran mosaik.
Pasien menjalani bronkoskopi dengan bronchoalveolar lavage. Cairannya berwarna putih
susu. Pemeriksaan sitopatologi menunjukkan adanya debris amorf dengan periodic acid-
Shiff (PAS): makrofag positif. Apakah diagnosis pasien ini?
A. Bronkiolitis obliteran – pneumonia
B. Pneumonitis interstitial deskuamatif
C. Nocardiosis
D. Pneumositis carinii pneumonia
E. Proteinosis alveolar pulmoner

78. Apakah penanganan yang tepat pada pasien di atas?


A. Doksisiklin
B. Prednison
C. Prednison dan siklofosfamid
D. Trimethoprim-sulfametoxazol
E. Lavage seluruh paru dengan normal saline

79. Laki-laki usia 68 tahun datang untuk evaluasi dispnea saat beraktivitas. Keluhan tersebut
disadari pasien pertama kali sekitar 2 tahun yang lalu. Pada saat itu, pasien sampai harus
berhenti berjalan di lapangan golf dan harus menggunakan golf cart, tetapi pasien masih
dapat menyelesaikan seluruh 18 holes. Selama setahun terakhir, pasien berhenti bermain
golf karena sesak napas yang ia rasakan tersebut. Pasien juga mulai susah untuk berjalan
dari dan ke kotak surat rumah pasien, yaitu sekitar 45 meter dari rumahnya. Pasien juga
mengeluhkan batuk kering hampir sepanjang hari. Batuk dikatakan tidak memburuk pada
malam hari, dan pasien mengatakan tidak ada yang mencetuskan keluhan tersebut. Bersin
atau pilek disangkal. Demam, merinding, atau pun penurunan berat badan juga disangkal
oleh pasien. Keluhan pada persendian dikatakan tidak ada. Riwayat merokok (+) sekitar 50
pak per tahun, tetapi pasien sudah berhenti merokok sejak 8 tahun lalu ketika pasien
didiagnosis dengan penyakit arteri koroner. Pemeriksaan fisik ditemukan pasien sesak napas
setelah berjalan di lorong ruang pemeriksaan, tetapi pasien dapat cepat pulih dengan
istirahat. Tanda vital: tekanan darah 118/67 mmHg, denyut jantung 88x/menit, laju
pernapasan 20x/menit, saturasi oksigen 94% saat istirahat dan turun jadi 86% setelah
berjalan 300 kaki. Pemeriksaan paru didapatkan perkusi dan ekspansi dada normal, adanya
Velcro-like crackles pada kedua basal paru yang terdistribusi setengah melalui kedua lapang
paru, wheezing (-). Pemeriksaan kardiovaskular dalam batas normal. Digiti clubbing (+).
Hasil CT scan thorax pasien seperti terlihat di bawah ini.

Kemudian, pasien dirujuk untuk dilakukan biopsi surgikal paru. Manakah gambaran
patologis yang paling mungkin ditemukan pada pasien ini?
A. Cairan amorf pekat dalam alveoli yang memberikan pewarnaan positif pada periodic
acid-shiff (PAS)
B. Destruksi alveoli dengan area emfisematosa, terutama pada lobus atas paru
C. Kerusakan alveolar difus
D. Pembentukan granuloma non kaseosa
E. Deposisi kolagen heterogen dengan fibroblast dan gambaran honeycomb

80. Beberapa kondisi di bawah ini yang dapat mencetuskan terjadinya bronkiektasis adalah,
kecuali…
A. Masuknya bahan hidrokarbon ke saluran napas
B. Infeksi Treponema pallidum
C. Pasien dengan asthmatic pulmonary eosinophilia
D. Aneurisma Rasmussen
E. Sindrom Kartagener

81. Sputum berbau busuk pada kasus bronkiektasis menandakan adanya infeksi sekunder oleh
karena…
A. Adenovirus tipe 21
B. Streptococcus pneumoniae
C. Treponema vincenti
D. Virus campak
E. Klebsiella ozaena

82. Pembedahan merupakan salah satu penanganan pasien bronkiektasis. Manakah di bawah ini
yang merupakan indikasi pembedahan pada kasus bronkiektasis?
A. Kasus bronkiektasi dengan PPOK
B. Bronkiektasis berat
C. Disertai bronchitis kronik
D. Bronkiektasis dengan komplikasi kor pulmonal kronik dekompensata
E. Bronkiektasis dengan hemoptisis massif

83. Pasien perempuan usia 56 tahun datang dengan keluhan batuk berhadak sejak 3 bulan yang
lalu. Dahak berwarna putih kekuningan, agak lengket, dan sulit keluar. Dahak dikatakan
sangat banyak terutama pada pagi hari setelah bangun tidur. Batuk berdahak terkadang
disertai darah. Keluhan disertai sesak napas yang dirasakan memberat 2 minggu belakangan
ini. Demam berulang sejak sebulan terakhir juga dirasakan oleh pasien. Pada pemeriksaan
paru ditemukan ronki kasar terutama di basal paru kanan, disertai wheezing minimal pada
paru kanan. Pasien direncanakan untuk melakukan pemeriksaan foto polos thorax. Apakah
yang Anda harapkan ditemukan pada foto polos thorax pasien ini?
A. Pergerseran mediastinum ke arah kiri
B. Honeycomb appearance
C. Hiperlusen paru
D. Air bronchogram (+)
E. Infiltrat pada bagian apex paru kanan disertai cavitas

84. Pasien perempuan usia 56 tahun datang dengan keluhan batuk berdahak sejak 3 bulan yang
lalu. Dahak berwarna putih kekuningan, agak lengket, dan sulit keluar. Dahak dikatakan
sangat banyak terutama pada pagi hari setelah bangun tidur. Batuk berdahak terkadang
disertai darah sedikit. Keluhan disertai sesak napas yang dirasakan memberat 2 minggu
belakangan ini. Demam berulang sejak sebulan terakhir juga dirasakan oleh pasien. Pada
pemeriksaan paru ditemukan ronki kasar terutama di basal paru kanan, disertai wheezing
minimal pada paru kanan. Pasien direncanakan untuk melakukan pemeriksaan foto polos
thorax. Apabila pasien ditemukan dengan foto polos thorax normal, maka tingkat derajat
penyakit pada pasien ini adalah…
A. Ringan
B. Sedang
C. Berat
D. Sangat berat
E. Mengancam jiwa

85. Pengobatan konservatif yang kurang tepat dilakukan pada pasien di atas adalah…
A. Pemberian antibiotik selama 7 – 10 hari
B. Perbaikan hidrasi tubuh
C. Mengatur posisi tidur pasien
D. Pemberian mukolitik
E. Menciptakan ruangan hangat dan udara lembab untuk pasien

86. Hal-hal berikut yang mempengaruhi keluhan pasien bronkiektasis antara lain, kecuali…
A. Ada atau tidaknya komplikasi
B. Tingkat derajat penyakit
C. Banyaknya bahan kimia korosif yang masuk ke saluran napas
D. Area bronkus yang terkena
E. Lokasi bronkus yang terkena

87. Diagnosis pasti bronkiektasis dapat ditegakkan melalui pemeriksaan…


A. Foto polos thorax
B. Bronkografi
C. Tes faal paru
D. MRI Thorax
E. CT scan thorax

1. Wanita 32 tahun datang ke instalasi gawat darurat, pasien saat ini sedang hamil 36 minggu,
pasien saat ini mengeluh sesak, pasien tidak ada mengeluhkan tentang kehamilanya dan
tidak ada masalah medis lainya. Pasien tidak minum obat-obatan kecuali vitamin untuk
kehamilan. Pada pemeriksaan pasien menunjukan penampilan dyspneic. TD: 128/78, nadi :
126 x/menit, rr : 28x/menit, saturasi O2 : 96% RA, pasien tidak demam. Pada pemeriksaan
paru dan jantung normal. Terdapat piting edema pada kedua tungkai. Pada pemeriksaan x-
ray paru normal. EKG normal irama sinus. Pada pemeriksaan Analisa gas darah
menunjukan Ph: 7,52, PaCO2: 26mmhg, PaO2 85mmhg. Apakah Langkah terbaik untuk
diagnosis dan managemen untuk pasien ?
A. Terapi inisial dengan amoxicillin untuk akut bronchitis
B. Melakukan pemeriksaan CT Paru angiogram
C. Melakukan Ecocardiogram
D. Menyakinkan pasien sesak saat kehamilan adalah normal dan tidak ada masalah yang
ditemukan pada pemeriksaan
E. Terapi dengan clonazepam untuk panic attack

89. Seorang laki-laki 22 tahun menderita cystic fibrosis. Pasien saat ini dirawat di rumah sakit
sekitar tiga kali setahun karena eksaserbasi infeksi. Pada pasien didapat kolonisasi kuman
Pseudomonas aeruginosa dan Staphylococcus aureus, tetapi tidak pernah memiliki
Burkholderia cepacian kompleks. pasien tetap aktif dan kuliah di jurusan arsitektur. Pasien
membutuhkan 2 L oksigen. Tes fungsi paru terbaru menunjukkan FEV1 yaitu 28% dari nilai
prediksi dan rasio FEV1/FVC sebesar 44%. Pengukuran gas darah arteri adalah pH 7,38,
PCO2 36 mmHg, dan PO2 62 mmHg. Manakah dari karakteristik berikut yang merupakan
indikasi rujukan untuk transplantasi paru?
A. Kolonisasi dengan Pseudomonas aeruginosa
B. FEV1 kurang dari 30% diprediksi
C. Rasio FEV1/FVC kurang dari 50%
D. PCO2 kurang dari 40 mmHg
E. Penggunaan oksigen dengan tenaga

90. Wanita 78 tahun dirawat di unit perawatan intensif medis dengan pneumonia multilobaris.
Pada awal masuk ke ruang gawat darurat, saturasi oksigen adalah 60% pada udara ruangan
dan hanya meningkat menjadi 82% Ketika memakai masker wajah non-rebreather. Pasien
mengalami gangguan pernapasan dan diintubasi di ruang gawat darurat. Setelah masuk ke
unit perawatan intensif, pasien dilakukan pembiusan. Ventilator diatur dalam mode assist-
control dengan laju pernapasan 24, volume tidal 6 mL/kg, FiO2 1,0, dan tekanan akhir
ekspirasi positif 12 cmH2O. Pengukuran gas darah arteri dilakukan pada pengaturan ini;
hasilnya adalah pH 7,20, PCO2 32 mmHg, dan PO2 54 mmHg. Apa penyebab hipoksemia?
A. Hipoventilasi saja
B. Hipoventilasi dan ketidaksesuaian ventilasi-perfusi
C. Shunt
D. Ketidakcocokan ventilasi-perfusi
E. Hiperventilasi

91. Seorang pria 65 tahun dievaluasi untuk keluhan sesak yang progresif pada aktivitas dan
batuk kering yang memburuk selama 6 bulan. Dia tidak mengalami sesak saat istirahat dan
menyangkal adanya suara mengik. Dia tidak mengalami nyeri dada. Dia memiliki riwayat
penyakit arteri koroner dan fibrilasi atrium, dan menjalani operasi bypass arteri koroner 12
tahun yang lalu. Obat yang dikonsumsi metoprolol, aspirin, warfarin, dan enalapril. Dia
sebelumnya merokok satu bungkus rokok setiap hari selama 40 tahun, berhenti 5 tahun
sebelumnya. Tanda-tanda vitalnya adalah tekanan darah 122/68, denyut jantung 68
kali/menit, frekuensi pernapasan 18 kali/menit, dan saturasi oksigen 92% pada udara
ruangan. Pemeriksaan dada menunjukkan ronki bibasilar muncul sekitar sepertiga atas
bilateral. Tidak terdengar mengi. Dia memiliki ritme yang tidak teratur dengan murmur
holosistolik II/VI di apeks. Tekanan vena jugularis tidak meningkat. Tidak ada edema,
tetapi ditemukan adanya clubbing. Tes fungsi paru mengungkapkan volume ekspirasi paksa
dalam 1 detik diprediksi 65%, kapasitas vital paksa diprediksi 67%, rasio FEV1/FVC 74%,
kapasitas paru total 68% diprediksi, dan kapasitas difusi karbon monoksida diprediksi 62% .
Tes mana yang paling mungkin untuk menentukan etiologi sesak pasien?
A. Bronkoskopi dengan biopsi paru transbronkial
B. CT angiografi paru
C. Ekokardiografi
D. CT scan dada beresolusi tinggi
E. Tes stres kedokteran nuklir

1. Pasien laki-laki usia 60 tahun datang ke UGD RS diantar oleh keluarganya oleh karena tiba-
tiba tidak sadarkan diri setelah memarahi anaknya. Sebelumnya pasien dikatakan mengeluh
nyeri kepala berat, muntah-muntah, berbicara pelo dan kelemahan separuh tubuh. Di RS
pasien tidak menunjukan perbaikan kesadaran, Setelah dirawat di RS beberapa hari keluarga
pasien mengatakan pasien demam. Pada pemeriksaan fisik td : 150/80 N: 90x/menit, RR;
25x/menit, T: 38,6, SpO2 : 88-89%, rongki +/+ , wh-/- ves+/+, kemudian pasien dilakukan
pemeriksaan lab. Wbc:16.000, HB:10,3, PLT : 230.000 rontgen thorax ap ditemukan
gambaran infiltral di hampir seluruh lapangan paru, apakah doiagnosis yang sesuai dengan
kasus diatas ?
A. Pneumonia Aspirasi
B. Community Acquired Pneumonia
C. Hospitalized Ascociated Pneumonia
D. Health Care Ascociated Pneumonia
E. PPOK eksaserbasi Akut

2. Dari kasus diatas apakah kuman tersering yang menyebabkan kondisi tersebut :
A. E.Coli
B. Enterobacter
C. Streptococcus
D. Bakteri anerob
E. Pseudomonas

3. Pasien laki-laki usia 70 tahun datang ke UGD RS diantar oleh keluarganya oleh karena tiba-
tiba lemah separuh tubuh, berbicara pelo dan kelemahan separuh tubuh. Pada pemeriksaan
fisik td : 150/80 N: 90x/menit, RR; 25x/menit, T: 38,4 0C SpO2 : 89-90, rongki +/+ , wh-/-
ves+/+, kemudian pasien dilakukan pemeriksaan lab. Wbc:17.000, HB:10,93 PLT : 230.000
rontgen thorax ap ditemukan gambaran infiltral di hampir seluruh lapangan paru, dokter
mendiagnosa sebagai Pneumonia Aspirasi, apakah pencegahan kasus diatas ? Kecuali
A. Posisi tidur setengah duduk dengan suudt 30-450
B. Prolaktis gigi dan menjaga hygiene oral
C. Fisioterapi dan pengaturan diet untuk pasien gangguan menelan
D. Antibiotic profilaksis
E. Menggunakan antacida dan Antihistamin H2

4. Dari kasus diatas dokter berencana untuk melakukan pemeriksaan penunjang selanjutnya
untuk dapat menentukan terapi definitive yang sesuai untuk pasien ?
A. C Reaktif Protein
B. Procalcitonin
C. Analisis Gas Darah
D. Kultur sputum sensitifitas
E. CT-Scan

5. Pasien 70 tahun datang memeriksakan kondisinya kedokter diantar oleh petugas panti werda.
datang dengan keluhan batuk kadang-kadang, sesak dan demam, dirumah pasien sudah
tidak bisa melakukan ADL (activity daily living ) dan pasien bergantung dengan petugas
pantai werda. Pada pemeriksaan fisik td : 150/80 N: 90x/menit, RR; 25x/menit, T: 37,4 0C
SpO2 : 92%, rongki +/+ , wh-/- ves+/+, kemudian pasien dilakukan pemeriksaan lab.
Wbc:16.000, HB:10,8 PLT : 200.000 rontgen thorax ap ditemukan gambaran infiltrate, dari
kasus tersebut apakah pencegahan yang disarankan untuk mencegah pasien mengalami hal
yang sama dikemudian hari :
A. Edukasi keluarga dan pasien untuk tidak hanya berbaring di tempat tidut dan tidur
dalam posisi duduk
B. Higine gigi geligi
C. Vaksinasi pneumokokus atau influenza
D. Pemberian antibiotic profilaksis dirumah
E. A+B+C benar
6. Dari kasus diatas apakah mikroorganisme yang paling sering menyebabkan terjadinya kondisi
diatas :
A. H. Influenza
B. Streptococcus Pneumonia
C. Staphylococcus Pneumonia
D. Kuman gram negative
E. Mycoplasma Pneumonia

7. Apakah faktor resiko yang dapat meningkatkan kejadian pneumonia aspirasi :


A. Miastenia gravis
B. Gastro esofageal reflux disease
C. Periodontitis
D. Semua benar
E. Semua salah

8. Pasien perempuan usia 30 tahun datang dengan keluhan batuk tanpa dahak & sesak sejak 2
minggu yang lalu, pasien merasa lemas dan badan terasa nyeri, kadang keluar keringat
malam, mengigil dan nyeri pada dadanya, pasien juga kadang-kadang mengalami demam.
Keluhan saat ini dirasakan makin memberat. Pasien mengatakan memiliki Riwayat bentol-
bentol Ketika udara dingin dan bersin-bersin ketika menghirup debu atau menghirup
parfum. Pada pemeriksaan fisik ditemukan adanya rongki yang okasional. Pada
pemeriksaan laboratorium ditemukan WBC : 12.0 x 10 3 / uL, HB : 14,0 g/dl, neutrophil 2,5x
103 / uL, Monosit 0,69x 103 / uL, Eosinofil 3x103 / uL, Basofil 0,05x103 / uL, PLT: 200.000.
pada pemeriksaan rontgen thorax ditemukan adanya gambaran infiltrate pada lapangan paru
dan kerley B-Lines. Dari data diatas apakah diagnose yang paling memungkinkan
A. Pneumonia aspirasi
B. Pneumonia komunitas
C. Pneumonia Eosinofilik
D. Health care associated pneumonia
E. Hospitalized associated pneumonia

9. Seorang pria berusia 55 tahun datang ke rumah sakit layanan primer dengan riwayat batuk
dan demam sejak 2 hari. Batuk penderita dikatakan produktif dengan dahak kental berwarna
hijau tua. Penderita memiliki riwayat hiperkolesterolemia yang rutin diobati dengan
Rosuvastatin. Penderita tidak merokok dan umumnya cukup sehat. Penderita rutin berolah
raga beberapa kali dalam seminggu. Penderita tidak memiliki kontak dengan orang sakit
sebelumnya dan tidak ingat riwayat penggunaan antibiotik. Pada pemeriksaan fisik,
ditemukan tanda-tanda vital sebagai berikut: Suhu 38,9 C, tekanan darah 132/78 mmHg,
Nadi 87 kali per menit, RR 20 kali per menit, dan SaO2 95% di udara ruangan. Pada
pemeriksaan fisik paru ditemukan suara nafas crackles di paru kanan dengan egophony.
Pada pemeriksaan rontgen dada menunjukkan konsolidasi segmental lobus kanan bawah
dengan air bronchograms. Apa pendekatan yang paling tepat untuk tatalaksana selanjutnya
dari pasien ini?
A. Ambil kultur dahak dan tunggu hasilnya sebelum memulai pengobatan.
B. Lakukan CT Scan thorax untuk menyingkirkan post obstruktif radang paru-paru.
C. Rujuk ke bagian gawat darurat untuk dirawat inap dan diberikan pengobatan dengan
antibiotik intravena.
D. Obati dengan doksisiklin 100 mg dua kali sehari.
E. Obati dengan moksifloksasin 400 mg setiap hari.
10. Seorang Wanita berusia 65 tahun dirawat di ruang ICU untuk manajemen syok septik akibat
kateter hemodialisis yang terinfeksi. Penderita awalnya diintubasi pada hari pertama di
rumah sakit dengan diagnosis sindrom gangguan pernafasan akut (ARDS). Penderita
perlahan-lahan membaik hingga FIO diturunkan menjadi 0,40. Penderita juga tidak demam
lagi atau membutuhkan vasopressor. Pada hari ke 7 di rumah sakit, penderita tiba-tiba
mengalami demam episode baru hingga 39,4C diikuti dengan keluarnya dahak berwarna
kuning kehijauan dari tabung endotrakealnya. Anda mencurigai pasien mengalami
pneumonia terkait ventilator. Apa cara terbaik untuk membuat diagnosis definitif pada
kasus ini?
A. Aspirasi endotrakeal menghasilkan organisme baru yang khas dari pneumonia terkait
ventilator.
B. Adanya infiltrat baru pada rontgen dada.
C. Kultur kuantitatif dari aspirasi endotrakeal menghasilkan lebih dari 106 organisme khas
dari pneumonia terkait ventilator.
D. Kultur kuantitatif dari spesimen sikat yang dilindungi menghasilkan lebih dari 103
organisme khas dari pneumonia terkait ventilator.
E. Tidak ada satu kriteria yang tepat untuk mendiagnosis pasien pneumonia terkait
ventilator.

11. Manakah dari pasangan berikut yang benar mengenai hubungan skenario klinis dan
pneumonia yang didapat dari masyarakat?
A. Pneumonia Aspirasi: Streptococcus Pyogenes
B. Penggunaan alkohol berat: Patogen atipikal dan Staphylococcus aureus
C. Kebersihan gigi yang buruk: Chlamydia Pneumoniae, Klebsiella Pneumoniae
D. Penyakit paru struktural: Pseudomonas aeruginosa
E. Perjalanan ke barat daya Amerika Serikat: Aspergillus Spp

12. Manakah dari berikut ini yang merupakan penyebab paling umum dari bronkiektasis difus
di seluruh dunia?
A. Fibrosis Kistik
B. Defisiensi immunoglobulin
C. Infeksi Mycobacterium Avium Intracellulare
D. Infeksi Mycobacterium Tuberculosis
E. Rheumatoid Arthritis

104.Seorang laki-laki, 45 tahun, datang ke UGD dengan keluhan sesak nafas sejak 30 menit
sebelum masuk rumah sakit. Nafas dikatakan berbunyi “ngik-ngik”. Penderita juga
mengeluhkan batuk berdahak yang kental dan sesekali terdapat bercak darah pada
dahaknya. Penderita memiliki riwayat penyakit asma yang sudah dideritanya sejak remaja
dan rutin menggunakan inhaler. Pasien sehari-harinya merokok namun dikatakan hanya 1
bungkus per hari. Pada pemeriksaan fisik ditemukan TD 140/90, Nadi 100 kali per menit,
RR 28 kali per menit, SaO2 90% udara ruangan. Pada pemeriksaan foto thorax ditemukan
gambaran fibrosis paru. Pada pemeriksaan laboratorium ditemukan peningkatan IgE dan
eosinophilia meningkat dalam darah. Pada pemeriksaan tes kulit ditemukan hasil positif A.
Fumigatus. Berdasarkan kasus di atas diagnosis yang tepat pada penderita adalah
A. Kandidosis
B. Mukormikosis
C. Allergic Bronchopulmonary Aspergillos (ABPA)
D. Histoplasmosis
E. TB Paru
105.Pada penyakit paru jamur akibat Aspergilloma ditemukan beberapa hal yang benar dibawah
ini, kecuali
A. Diagnosis Aspergilloma umumnya ditegakkan berdasarkan pemeriksaan radiologis
yaitu adanya gambaran khas berupa fungus ball.
B. Lebih banyak terjadi pada pasien yang sudah mempunyai kelainan anatomis pada paru
misalnya karena TB Paru, abses paru, dan tumor paru.
C. Secara radiologis akan terlihat kelompok hifa dan spora jamur yang memberikan
bayangan radioopak sedangkan rongga kavitas radiolusen.
D. Penegakkan diagnosis berdasarkan kriteria ditemukan riwayat asma dan eosinophilia
dalam darah yang meningkat.
E. Gejala utama dan mengancam nyawa adalah batuk darah (hemoptisis)

106.Seorang perempuan 38 tahun di rawat di ruang intensif dengan penurunan kesadaran. Pasien
dirawat dengan terpasang alat intubasi endotrakeal, oksigen NRM, selang NGT, dan Dower
Kateter. Penderita baru diketahui memiliki infeksi HIV AIDS 3 bulan yang lalu dan
menolak untuk berobat. Saat 3 hari di rawat inap, nafas penderita terdengar seperti
mengorok dan ditemukan banyak dahak ketika dilakukan suction berkala. Pada pemeriksaan
rontgen dada menunjukkan adanya gambaran efusi pleura. Pada pemeriksaan kultur dahak
10% akan ditemukan adanya?
A. Aspergilosis
B. Cryptococcus Neoformans
C. Histoplasmosis
D. Pneumocystis Pneumonia
E. Pneumocystis Carinii

107.Pernyataan berikut ini benar mengenai Penyakit Paru Jamur Histoplasmosis kecuali
A. Patogen penyebab tersering adalah H. Capsulatum.
B. Masa inkubasi sekitar 14 hari dengan gambaran klinis terkadang menyerupai
tuberculosis.
C. Histoplasmosis Asimtomatik tidak menimbulkan gejala walaupun tes histoplasmin
positif.
D. Histopasmosis Diseminata sering terjadi pada pasien Limfoma Hodgkin.
E. Penyakit Paru Jamur akibat Histoplasmosis dibagi menjadi Histoplamosis asimtomatik,
Histoplasmosis paru akut, dan Histoplasmosis Diseminata.

108.Tatalaksana pada Penyakit Paru Jamur dibawah ini yang tepat, kecuali
A. Aspergiloma tidak memerlukan pengobatan tetapi bila terjadi batuk darah yang hebat
dengan cadangan fungsi paru cukup memadai dapat dilakukan operasi berupa reseksi
paru.
B. Lama pengobatan yang optimal untuk aspergilosis invasif idelanya adalah 4 sampai 6
minggu menggunakan profilaksis itrakonasol oral 2 kali 200 mg sehari.
C. Untuk kandidosis paru diberikan Amfoterisin B IV 0,5 – 0,7 mg/kg./hari selama 2-4
minggu.
D. Untuk infeksi jamur yang tidak mengancam nyawa pilihan jatuh ke flukonasol.
E. Aspergilosis bronkopulmoner alergik dapat diberikan kortikosteroid oral.

109.Seorang laki-laki, 45 tahun, datang ke UGD dengan keluhan sesak nafas sejak 30 menit
sebelum masuk rumah sakit. Nafas dikatakan berbunyi “ngik-ngik”. Penderita juga
mengeluhkan batuk berdahak yang kental dan sesekali terdapat bercak darah pada
dahaknya. Penderita memiliki riwayat penyakit asma yang sudah dideritanya sejak remaja
dan rutin menggunakan inhaler. Pasien sehari-harinya merokok namun dikatakan hanya 1
bungkus per hari. Pada pemeriksaan fisik ditemukan TD 140/90, Nadi 100 kali per menit,
RR 28 kali per menit, SaO2 90% udara ruangan. Pada pemeriksaan foto thorax ditemukan
gambaran fibrosis paru. Pada pemeriksaan laboratorium ditemukan peningkatan IgE dan
eosinophilia meningkat dalam darah. Pada pemeriksaan tes kulit ditemukan hasil positif A.
Fumigatus. Berdasarkan kasus di atas staging diagnosis yang tepat pada penderita adalah
A. ABPA stage remisi
B. ABPA stage eksaserbasi ulang
C. ABPA stage fibrosis paru
D. ABPA stage refrakter
E. ABPA stage kronis

110.Pada pemeriksaan fisik seorang penderita Limfoma Hodgkin ditemukan gambaran


hepatosplenomegaly dan limfadenopati. Gejala klinis penderita mendadak demam tinggi
dan setelah dilakukan pemeriksaan darah lengkap ditemukan pansitopenia. Pada foto
rontgen dada terdapat gambaran infiltrat difus pada paru. Diagnosis yang tepat adalah
A. Histoplasmosis Diseminata
B. Aspergilloma
C. ABPA
D. Cryptococcus
E. Kandidosis

111.Pada Kandidosis dan Mukormikosis akan ditemukan hal berikut ini, kecuali?
A. Kandidosis paru dapat hidup sebagai saprofit di saluran nafas sedangkan Mukormikosis
dapat tersebar luas di alam
B. Pada Kandidosis kadang-kadang berupa misetoma sedangkan Mukormikosis harus
terdapat faktor predisposes berupa diabetes melitus dan penyakit kronis lainnya.
C. Kandidosis paru primer umumnya berasal dari ekstra paru sedangkan Mukormikosis
terjadi setelah inhalasi jamur kemudian terjadi thrombosis.
D. Kandidosis dapat menyebabkan nodul di paru berukuran 10 mm dan dapat bermanifest
ke ginjal, sedangkan Mukormikosis tidak dapat bermanifestasi ke luar paru.
E. Kandidosis dapat berada di saluran vagina sedangkan Mukosis disebabkan oleh
Nokoardiosis sp N. Astorides

112.Seorang wanita berusia 54 tahun datang dengan keluhan batuk kronis yang memburuk
sejak 6-12 bulan ini. Dia mengatakan batuk pada siang dan malam hari, batuk produktif
dengan dahak warna hijau dan kental. Jumlah dahak sekitar 100 ml setiap hari. Dijumpai
crakcles kasar bilateral pada paru bawah.tes fungsi paru menunjukkan FEV1 sebesar 1.68 L
(prediksi 53.3%), FVC sebesar 3 L (prediksi 75%), dan rasio FEV1/FVC sebesar 56%.
Radiografi toraks dalam batas normal. Apa yang akan anda rekomendasikan sebagai
langkah selanjutnya dalam evaluasi pasien ini?
A. Bronkoskopi dengan lavage bronchoalveolar
B. CT dada dengan kontras intravena
C. CT dada resolusi tinggi
D. Kadar immunoglobin serum
E. Pengobatan dengan bronkodilator kerja Panjang dan kortikosteroid inhalasi

113.Seorang laki-laki 48 tahun dirawat di rumah sakit dengan keluhan demam dan batuk. Dia
merupakan alkoholisme dan tunawisma. Dia tidak secara rutin mendapatkan perawatan
kesehatan. Dia sering merasa kelelahan dan malaise selama 8 minggu ini, kehilangan berat
badan namun tidak mengetahui berapa kehilangan berat badan, demam (+). Batuk dengan
produk sputum yang berbau, dengan dahak berwarna gelap dan kadang berdarah. Jumlah
produksi dahak sekitar 3 sendok makan sehari. Riwayat minum alkohol (+), merokok (+) 1
bungkus setiap hari. Pemeriksaan vital sign TD 110/73 mmHg, HR 98kali/menit, RR 20
kali/menit, T 38.2C, saturasi oksigen 94% dengan udara ruangan. Nafas bau (+). Bunyi
napas amforik terdengar di posterior lapangan paru kanan bawah. rontgen toraks
menunjukkan lesi paru kavitas 4cm dilobus kanan bawah. Pasien dirawat diruang isolasi.
Dilakukan pemeriksaan kultur sputum, mikobakteri, dan jamur. Apa pilihan awal terbaik
untuk terapi pada pasien ini?
A. Ampisilin -sulbaktam 3 gr setiap 6 jam (intravena)
B. Isoniazid, rifampisin, pirazinamid, dan etambutol secara oral
C. Metronidazole 500mg setiap 6 jam secara oral
D. Drainase cavitas perkutan
E. Piperacillin-tazobactam 2.25gr setiap 4jam dengan kombinasi tobramycin 5mg/kg
intravena setiap hari

114.Seorang laki-laki berusia 35 tahun datang ke klinik infertilitas, dia tidak memiliki anak dan
setelah 2 tahun berhubungan seksual dengan istri tanpa alat pelindung namun istri juga
belum hamil. Analisis sperma menunjukkan jumlah sperma normal, tetapi sperma tidak
bergerak. Riwayat medis sebelumnya infeksi sinopulmoner berulang, dan baru baru ini
mendeirta bronkiektasis. Radiografi dada cenderung menunjukkan yang mana dari berikut
ini:
A. Limfadenopati bihilar
B. Infiltrat lobus atas bilateral
C. Normal
D. Situs terbalik
E. Balon air berbentuk hati

115.Seorang wanita 28 tahun dievaluasi untuk infeksi paru dan sinus berulang. Dia memiliki
riwayat bronkitis sejak awal remaja dan mengatakan bahwa selama 5 tahun terakhir
mengkonsumsi antibiotik setidaknya tiga kali setahun. dia juga mengatakan kesulitan
menambah berat badan dan selalu merasa pendek dibandingkan teman-temannya. Pada
pemeriksaan fisik, IMT 18.5kg/m2. Saturasi oksigen 94% pada udara ruangan. Ada polip,
ronkhi kasar pada apeks paru bilateral, clubbing (+). Rontgen thorax menunjukkan
bronkiektasis lobus atas bilateral dengan area sumbatan lender. Anda khawatir tentang
kemungkinan cystic fibrosis yang tidak terdiagnosis. Manakah dari tes berikut ini yang
dapat mendukung diagnosis cystic fibrosis?
A. Analisis DNA menunjukkan satu Salinan alel delta F508
B. Penurunan perbedaan potensial hidung awal
C. Di jumpai pseudomonas aeruginosa pada kultur dahak berulang
D. Nilai klorida keringat > 35 meq/L
E. Nilai klorida keringat > 70 meq/L

116.Seorang pasien laki-laki berusia 39 tahun datang ke IGD dengan keluhan sesak napas yang
dialami sejak 2 hari yang lalu. Sesak terus- menerus dan tidak dipengaruhi aktivitas. Pasien
merupakan seorang pekerja pabrik kopi selama 15 tahun. Pasien juga mengeluhkan batuk
berdahak sejak 2 hari yang lalu, demam disangkal. Riwayat penyakit asma disangkal,
Riwayat diabetes melitus disangkal, riwayat penyakit jantung dan ginjal disangkal. Dari
hasil pemeriksaan fisik, didapatkan TD 120/70mmHg, HR 92 kali/menit, RR 24 kali/menit,
t 36.5c. Pada pemeriksaan fisik thorax didapatkan ronchi pada kedua basal paru. Pada hasil
pemeriksaan penunjang CT Scan thorax didapatkan nodular sentrilobular atau high
attenuation pada area percabangan seperti gambaran lesi bronkiolar. Apa diagnosa pasien
ini?
A. Pneumokoniosis
B. Sarkoidosis
C. Bronchiolitis
D. Syndrome goodpasture
E. Mesotelioma

117.Manakah pernyataan yang benar dibawah ini terkait dengan sarkoidosis paru, kecuali
A. Merupakan pemyakit granulomatosa sistemik yang dapat mengenai semua organ
B. Pada sarkoidosis ada uji kulit K veim-stilzbach
C. Gejala tersering adalah batuk dan sesak napas
D. Sarkoidosis fibrokistik dapat berkomplikasi bronkiektasis, misetoma dan hemoptisis
E. Pengobatan dengan prednisone 45mg/hari selama 2 minggu lalu ditapering off jadi
10mg/hari selama 2 minggu

118.Seorang pasien laki-laki berusia 42 tahun dengan keluhan sesak napas sejak 4 hari yang
lalu, pasien juga mengeluhkan nyeri dada, demam, dan mudah lemas. Riwayat penyakit
jantung disangkal. Pasien merupakan seorang pekerja pabrik pertambangan sudah 10 tahun.
Dari hasil pemeriksaan penunjang radiografi toraks didapatkan gambaran opasitas nodular
pada lapangan paru atas yaitu membentuk eggshell . kemungkinan diagnosa pasien ini
adalah
A. Asma kerja
B. Bisinosis
C. Potter”s rot
D. TB paru
E. PPOK

119.Dalam menegakkan diagnosa pneumonitis hipersensitivitas, ada beberapa kriteria mayor


dan minor. yang termasuk dalam kriteria mayor adalah
A. Ronki ke 2 basal paru
B. Limfositosis dari cairan lavase bronkus
C. Kapasitas difusi paru menurun
D. Ada bukti paparan antigen yang sesuai, baik dari anamnesis maupun pemeriksaan
antibodi serum
E. Hipoksemia arteri, baik karena latihan atau saat istirahat

120.Penyakit dibawah ini yang dapat disebabkan oleh asbes yaitu


A. Plak pleura
B. Mesothelioma maligna
C. Efusi pleura
D. A dan C benar
E. A, B, dan C benar

121.Seorang wanita berusia 25 tahun dengan keluhan sesak napas dialami sejak 1 hari yang lalu,
gejala sesak memberat pada malam hari. Pasien juga mengeluhkan terasa berat pada dada
dan batuk. Pemeriksaan vital sign didapatkan TD 110/70mmHg, HR 84 kali/menit, RR 24
kali/menit, T 36.8 C. Pasien sudah bekerja di pabrik tekstil sejak 6 tahun yang lalu. Terapi
awal yang dapat diberikan kepada pasien ini adalah
A. Injeksi metilprednisolon
B. Prednisone tablet
C. Albuterol inhalasi
D. Injkesi dipenhidramin
E. Salmeterol

122. Kemungkinan diagnosa pasien diatas adalah


A. Antrakosis
B. Bisinosis
C. Bronkitis
D. PPOK
E. Silikosis

123.Seorang pasien datang dengan keluhan batuk tidak berdahak, sesak yang progresif, ronkhi
kering pada akhir inspirasi (terutama di basal paru), didapatkan jari tabuh, sianosis dan
bunyi jantung ke 2 dari katup pulmonalis jantung mengeras. Pasien telah terdiagnosa
menderita cryptogenic fibrosing alveolitis. Manakah pengobatan yang dapat diberikan
pada pasien tersebut?
A. Kortikosteroid
B. Imunosupresan
C. Kolkisin
D. Semua diatas benar
E. Semua diatas salah

124.Seorang pria 22 tahun dengan cystic fibrosis terlihat untuk pemeriksaan lanjutan rutin. Dia
saat ini dirawat dengan DNAse manusia rekombinan dan albuterol dengan nebulisasi dua
kali sehari. Teknik pembersihan dahak utamanya adalah latihan aerobik lima kali seminggu
dan drainase autogenik. Dia merasa baik secara keseluruhan, dan pemeriksaannya normal.
Pengujian fungsi paru menunjukkan FEV1 sebesar 4,48 L (diperkirakan 97%), FVC sebesar
5,70 L (diprediksi 103%), dan rasio FEV1/FVC sebesar 79%. Kultur dahak rutin
menumbuhkan Pseudomonas aeruginosa. Satu-satunya organisme yang diisolasi pada kultur
sebelumnya adalah Staphylococcus aureus. Apa yang Anda rekomendasikan untuk pasien
ini?
A. Osilasi dinding dada frekuensi tinggi
B. Saline hipertonik (7%) nebulisasi dua kali sehari
C. Tobramycin inhalasi 300 mg dua kali sehari setiap bulan
D. Cefepime dan tobramycin intravena selama 14 hari
E. Kunjungan kembali dalam 3 bulan dengan biakan dahak berulang dan pengobatan hanya
jika ada P. aeruginosa yang persisten

125.Berikut yang tidak mungkin ditemukan dalam dahak pasien dengan cystic fibrosis?
A. Haemophilus influenzae
B. Acinetobacter baumannii
C. Burkholderia cepacia
D. Aspergillus fumigatus
E. Staphylococcus aureus

126.Berikut ini adalah faktor risiko penyakit paru obstruktif kronik kecuali:
A. Hiperresponsif saluran napas
B. Paparan debu batu bara
C. Paparan asap rokok pasif
D. Infeksi saluran pernapasan berulang
E. Penggunaan bahan bakar biomassa di area yang berventilasi buruk

127.Seorang wanita 65 tahun dievaluasi untuk dyspnea saat aktivitas dan batuk kronis. Dia
memiliki sejarah panjang penggunaan tembakau, merokok 1,5 bungkus setiap hari sejak
usia 20 tahun. Dia adalah seorang wanita kurus tanpa kesulitan yang jelas. Saturasi
oksigennya di udara ruangan adalah 93% dengan laju pernapasan 22/menit. Paru-paru
mengalami hiperekspansi pada perkusi dengan penurunan suara nafas pada lapang paru
bagian atas. Anda mencurigai penyakit paru obstruktif kronik. Apa temuan yang diharapkan
pada tes fungsi paru ?

FEV1 FVC FEV1/FVC TLC DLCO


Ratio
A Menurun Normal atau menurun Menurun Menurun Menurun
B Menurun Normal atau menurun Menurun Meningkat Menurun
C Menurun Menurun normal Menurun Menurun
D Menurun Normal atau menurun Menurun meningkat Normal atau
meningkat
E Meningkat Normal atau menurun Menurun Menurun Normal atau
meningkat

128. Apa dasar untuk mengindentifikasi spesies kuman BTA adalah kecuali
A. Waktu pertumbuhan
B. Pembentukan pigmen
C. Tes biokimia
D. Suhu pertumbuhan
E. Pewarnaan

129.Menurut ernest runyon dapat membedakan golongan mikrobakterium atipik menjadi


beberapa Grup kecuali :
A. Grup I ( fotokromogen )
B. Grup II ( Skotokromogen )
C. Grup IV ( rapid growers )
D. Grup V ( non fotokromogen )
E. Grup III ( non fotokromogen )

130.Untuk membunuh dan mengendalikan infeksi kuman mikobakterium didalam sel imun
selular yang berperan adalah kecuali
A. Proliferasi CD4+
B. Limfosit T
C. Pelepasan g interferon
D. Interleukin 12
E. Interleukin 6

131.Pada pasien AIDS yang mendapat infeksi Mikobakterium Avium – Intraselular diseminata
( ATS 1997 ) anjuran terapinya adalah kecuali
A. Klaritromisin (500 mg 2 kali sehari )
B. Azitromisin 500mg sehari
C. Etambutol 15mg/kg sehari
D. Rifabutin ( 150-300 mg ) sehari.
E. etambutol 30 mg/kg sehari

132.Menurut ATS (1997 ) berapa lama pengobatan infeksi AIDS dengan MAI dengan jumlah
set T CD4 > 100/ml sewaktu mendapatkan pengobatan HAART adalah
A. >2 bulan
B. >3 bulan
C. >4 bulan
D. >5 bulan
E. > 6 bulan

133.Kuman yang resisten terhadap terapi OAT standar adalah


A. Mikobakterium Fortuitum dan mikobakterium tuberkulosis
B. Mikobakterium chelonae dan mikobakterium tuberculosis
C. Mikrobaketrium tuberculosis dan Mikobakterium Avium – Intraselular
D. Mikobakterium Fortuitum dan Mikobakterium chelonae
E. Mikobakterium kansasii dan Mikobakterium Avium – Intraselular

134.Terapi antibiotic yang sensitive terhadap kuman mikobakterium Fortuitum adalah kecuali
A. Amikasin
B. Siprofloksasin
C. Sefositin
D. Imipenem
E. Peniciline

135.Golongan kuman Mikobakterium atipik menurut ernest Runyon yang warna koloni tidak
dipengaruhi oleh cahaya adalah
A. Grup I ( fotokromogen )
B. Grup II ( Skotokromogen )
C. Grup IV ( rapid growers )
D. Grup V ( non fotokromogen )
E. Grup III ( non fotokromogen )

136.Seorang pria 72 tahun dengan keluhan sesak napas progresif saat beraktivitas sejak 3
minggu yang lalu. Pasien batuk produktif ringan dan anoreksia tapi menyangkal adanya
demam, menggigil, dan berkeringat. Pada pemeriksaan fisik, dengan tanda-tanda vital
normal dan saturasi oksigen normal pada udara ruangan. Tekanan vena jugularis normal
dengan pemeriksaan suara jantung menjauh tetapi tidak ada kelainan lain. Trakea terletak
pada garis tengah dan tidak ada limfadenopati. Pada pemeriksaan paru kiri tertinggal,
fremitus taktil menurun, didapatkan suara redup pada seluruh lapangan paru kiri, suara
napas menurun. Pemeriksaan paru-paru kanan normal. Setelah X-Ray dada, apakah
tatalaksana awal yang tepat pada pasien ini?
A. Antibiotik intravena
B. Thoracentesis
C. Bronkoskopi
D. CT Scan dada
E. Bronkodilator
137. Penyebab effusi pleura paling sering adalah
A. Malignansi
B. TB
C. hydrothorax hepatik
D. CHF
E. Pneumonia akibat bakteri

138. Apakah treatment of choice dari spontaneous bacterial pleuritis:


A. Observasi
B. Hidrasi
C. Kortikosteroid
D. Antibiotik
E. Kultur dan uji sensitivitas cairan pleura

139. Seorang pasien laki-laki 55 tahun datang ke UGD oleh karena bengkak dan sesak. Pasien
memiliki riwayat sindrom nefrotik yang terkadang mengalami eksaserbasi akut. Hasil
pemeriksaan radiologis dan penunjang lain menunjukkan effusi pleura eksudatif. Diagnosis
apakah yang harus ditegakkan ketika berhadapan dengan pasien sindrom nefrotik yang
datang dengan effusi pleura eksudatif?
A. Empyema
B. Emboli paru
C. Kanker paru
D. Mesothelioma
E. Tuberkulosis

140. Manakah pernyataan mengenai thorakosentesis berikut yang benar dibawah ini
A.Transfusi FFP direkomendasikan sebelum prosedur untuk mencegah kemungkinan
perdarahan
B. Sensitivitas diagnositik sitologi cairan untuk kasus effusi pleura akibat malignansi
mencapai 60-80%
C. Terdapatnya transudat dapat mengeksklusi kemungkinan effusi pleura malignan
D. Apabila sitologi cairan pleura tidak menegakkan diagnostic (nondiagnostic), biopsi
pleura torakoskopi harus dilakukan selanjutnya
E. Komplikasi perdarahan pada pasien dengan gangguan faktor koagulan mencapai 25-
40%.

141.Seorang laki-laki 69 tahun tanpa riwayat penyakit signifikan datang dengan keluhan batuk
produktif sejak 5 hari yang lalu, dengan karakteristik sputum kuning dengan sesekali
mengandung bercak darah. Demam dan menggigil (+). Pasien merupakan perokok ringan
dan bekerja sebagai pengemudi limusin. Tanda-tanda vital: nadi 104 kali/menit, frekuensi
pernapasan 22 kali/menit, dan saturasi oksigen 95% pada oksigen melalui nasal cannule 2
L/menit. Pasien tampak sesak dan mengeluarkan banyak keringat. Pemeriksaan paru
menunjukkan suara napas yang berkurang pada thorax lobus inferior dengan suara redup
pada perkusi, tanpa disertai egofoni. Pemeriksaan penunjang: leukosit 26.000 sel/μL. LDH
139 U/L (normal 90-225 U/L), protein total serum 6,6 g/dL (kisaran normal 6,5-8,5 g/dL),
dan GDS 81 g/dL. dL (kisaran normal 74–110 g/dL). Rontgen thorax dan CT-Scan pasien
terlampir. Hasil berikut diperoleh pada thoracentesis:

Parameter Hasil
Warna Kuning
Jumlah sel 660 sel/µL dengan neutrogfil 88%
Ph 7,08
LDH 1,248 U/L
Protein total 4,6 g/Dl
Glukosa 25 g/dL
Gram Gram (+) coccus berpasangan berbentuk rantai
Hasil rontgen thorax AP Hasil CT-Scan

Pernyataan dibawah berikut yang benar mengenai diagnosis dan terapi pada kasus ini
adalah:
A. Diagnosis: Simple parapneumonic effusion, terapi: thorakotomi
B. Diagnosis: Simple parapneumonic effusion, terapi: VATS
C. Diagnosis: Complicated parapneumonic effusion, terapi: antibiotic
D. Diagnosis: Complicated parapneumonic effusion, terapi: VATS
E. Diagnosis: Empyema, terapi thorakotomi

142. Manakah dari pilihan etiologi berikut yang tidak berkaitan dengan chylothorax:
A. Kaposi’s sarcoma
B. Trauma
C. Idiopathic pulmonary fibrosis
D. Lymphangoleiomyomatosis
E. Limfoma

143. Manakah dari pilihan berikut yang merupakan penyebab tersering hemothorax?
A. Iatrogenik
B. Antikoagulan
C. Trombositopeniaa
D. Trauma
E. Endometriosis

144. Seorang wanita 40 tahun dengan riwayat asma persisten derajat sedang telah terkontrol
dengan baik selama 3 bulan terakhir dan saat ini menggunakan albuterol Metered Dose
Inhaler (MDI) sekali seminggu untuk kontrol gejala. Pasien terbangun di malam hari
sejumlah dua kali setiap bulan dengan gejala asma, pasien masih mampu terus beraktivitas
dan berolahraga secara teratur tanpa kesulitan. Pasien saat ini juga mengkonsumsi
fluticasone inhalasi 88 g/puff dua kali sehari dan salmeterol 50 g dua kali sehari. Hasil
pemeriksaan FEV1 saat ini 83% dari personal best. Apakah Langkah selanjutnya yang
paling tepat untuk saat ini?
A. Tambahkan montelukast 1x10 mg, karena penggunaan albuterol saat ini menunjukkan
kontrol asma yang buruk.
B. Turunkan dosis flutikason hingga 44 g/puff, dua kali sehari.
C. Hentikan flutikason.
D. Hentikan salmeterol.
E. Tidak melakukan apa-apa, karena penggunaan albuterol saat ini menunjukkan kontrol
asma yang buruk.

145.Seorang dokter sedang mempertimbangkan terapi omalizumab untuk pasien dengan asma
persisten berat yang membutuhkan prednison oral 5-10 mg setiap hari, kortikosteroid
inhalasi dosis tinggi, bronkodilator long-acting, dan montelukast untuk mengontrol
gejalanya. Manakah dari pilihan berikut ini yang diperlukan/harus dilakukan sebelum
memulai omalizumab?
A. Penghentian prednison oral
B. Data peningkatan kadar IgE hingga lebih dari 1000 IU/L
C. Normalisasi FEV1 atau laju PEF
D. Riwayat sensitivitas terhadap aeroallergen
E. Penggantian prednison oral ke prednisolon intravena

146. Seorang wanita 76 tahun dievaluasi untuk sesak napas akut dan batuk kering selama 2 hari
terakhir. Pasien mengeluh demam. Riwayat penyakit dahulu: hipotiroidisme dan diabetes
mellitus. Pasien saat ini mengkonsumsi metformin 1000 mg, dua kali sehari, levothyroxine
yang dosisnya ditingkatkan menjadi 100 g setiap hari, sejak 1 bulan yang lalu, dan
nitrofurantoin 100 mg, dua kali sehari, sejak 3 hari yang lalu untuk infeksi saluran
kemih. Tanda-tanda vital: suhu 39,2°C, tekanan darah 115/82, nadi 96 kali per menit, laju
pernapasan 24 kali per menit, suhu 38,5°C, dan SpO 2 94% pada udara kamar. Pada
pemeriksaan fisik ditemukan perkusi redup dan penurunan suara nafas pada regio dasar paru
kanan. Crackles bilateral paru (+). Radiografi thorax menunjukkan efusi pleura sisi kanan
derajat sedang dan infiltrat paru tipe patchy bilateral. Thoracentesis menunjukkan efusi
eksudatif dengan jumlah sel darah putih 3500/mm3 dengan differensial sel PMN 60%,
eosinophil 30%, dan limfosit 10%. Hasil bronkoskopi menunjukkan diferensial sel PMN
50%, eosinophil 15%, dan alveolar makrofag 35%. Manakah dari berikut ini yang akan
menjadi langkah selanjutnya dalam penatalaksaan pasien ini?
A. Menunggu hasil kultur cairan pleura sebelum membuat rekomendasi pengobatan.
B. Turunkan dosis levothyroxine.
C. Hentikan nitrofurantoin.
D. Meningkatkan dosis levothyroxine.
E. Mulai pengobatan dengan terapi steroid dosis tinggi (metilprednisolon 1 g sehari).
147. Seorang wanita 34 tahun datang dengan keluhan batuk dan sesak saat aktivitas yang secara
memburuk selama 3 bulan terakhir. Pasien tidak memiliki riwayat penyakit paru
sebelumnya dan tidak pernah menderita asma. Pasien mulai bekerja di sebuah toko hewan
peliharaan sejak 6 bulan yang lalu. Pekerjaan pasien antara lain membersihkan sangkar
reptil dan burung. Pasien mengeluhkan demam ringan sesekali, batuk kering dan tidak
produktif, tetapi tidak mengeluhkan mengi. Sebelum 3 bulan yang lalu, pasien tidak
memiliki batasan toleransi aktivitas/exercise, tetapi sekarang pasien melaporkan bahwa
pasien mengeluh sesak saat menaiki dua tangga. Tanda vital: SpO2 95% pada room air saat
istirahat, tetapi desaturasi menjadi 89% Ketika beraktivitas/exercise. Suhu
37,7°C. Pemeriksaan paru dalam batas normal. Clubbing atau sianosis (-). Hasil rontgen
thorax normal. HRCT thorax menunjukkan infiltrat ground-glass difus di lobus bawah paru
dengan nodul sentrilobular. Biopsi transbronkial menunjukkan infiltrat alveolar interstitial
sel plasma, limfosit, dan eosinophil (eosinophil jarang) disertai granuloma nonkaseosa
longgar. Hasil kultur negatif untuk bakteri, virus, dan jamur patogen. Apa diagnosisnya?
A. Aspergillosis
B. Pneumonitis hipersensitivitas
C. Pneumonitis interstisial nonspesifik terkait dengan penyakit vaskular kolagen
D. Psittacosis
E. Sarkoidosis

148.Seorang laki-laki berusia 45 tahun dievaluasi di klinik untuk penyakit asma. Gejala dimulai
sejak 2 tahun yang lalu dan mengeluhkan batuk episodik, mengi serta pasien biasa
menggunakan bronkodilator inhalasi dan kortikosteroid inhalasi tetapi sekarang sudah tidak
begitu efektif lagi dan membutuhkan obat prednison. Pasien mengatakan bahwa gejalanya
bertambah berat pada hari kerja tetapi tidak dapat menentukan pemicu spesifik. Obat-obatan
yang dikonsumsi adalah albuterol MDI, fluticasone MDI, dan prednison 10 mg po setiap
hari. Pasien bekerja sebagai buruh tekstil. Pemeriksaan fisik terdapat mengi ekspirasi
polifonik difus ringan dan tidak ditemukan kelainan lain. Manakah dari berikut ini yang
langkah selanjutnya yang paling tepat?
A. Latihan tes fisiologi
B. Pengukuran FEV1 sebelum dan sesudah bekerja
C. Tes tantangan metakolin
D. Tes kulit untuk alergi
E. Kultur dahak untuk Aspergillus fumigatus

149. Sebutkan faktor – faktor predisposisi terjadinya abses paru?


A. Kartagener’s syndrome, immunosupresi, bronkiektasis, dan alkoholisme.
B. Kartagener’s syndrome, immunosupresi, bronkiektasis, dan pancreatic phlegmon.
C. Kartagener’s syndrome, immunosupresi, bacteriodes fragie, dan HIV.
D. Mikobacterium tuberculosis, immunosupresi, bronkiektasis, dan alkoholisme.
E. Staphilococus aureus, immunosupresi, bronkiektasis, dan alkoholisme.

150. Etiologi dari abses paru dapat disebabkan oleh mikrorganisme sebagai berikut?
A. Bacteriodes fragilis, clostridium perfringens, salmonella typhi, helicobacter pylori.
B. Bacteriodes fragilis, helicobacter pylori, salmonella typhy, pneumocystitis.
C. Bacteriodes fragilis, clostridium perfringens, klebsiella pneumonia, pneumocystitis.
D. Clostridium perfringens, klebsiella pneumonia, pneumocystitis, chlamydia trachomatis
E. Clostridium perfringens, klebsiella pneumonia, helicobacter pylori, pneumocystitis.
151.Wanita 45 tahun datang ke IGD dengan keluhan sesak napas, tidak ada demam, batuk
berdahak encer. Pasien sudah 1 tahun terakhir didiagnosis Siskemik sklerosis. Dari
pemeriksaan fisik ditemukan ronkhi di basal paru kanan. Pemeriksaanradiologi ditemukan
gambaran honey comb appearance. Kemungkinan kmplikasi pada pasien tersebut adalah:
A. Pneumonitis interstitial
B. Bronkiextasis terinfeksi
C. Interstitial Lung Disease
D. Hipertensi pulmonal
E. Tuberculosis paru

152.Laki-laki 22 tahun datang dengan keluhan sesak napas. Sesak napas semakin bertambah ,
diawali dengan nyeri dada kanan ketika menarik napas panjang, semakin bertambah bila
miring ke kiri dan berkurang bila miring ke kanan. Batuk (-), demam (-), kelainan BAK (-).
Pemeriksaan fisik: RR 24 x/i, hemithoraks kanan lebih cembung, tertinggal, sela iga
melebar, perkusi dullness, mulai sela iga III ke bawah, hipersonor sela iga III ke atas.
Diagnosa:
A. Pneumothoraks
B. Emfisema Paru
C. Bronchitis akut
D. Hidro pneumothorax
E. Atelektasis
153.Seorang penderita pneumonia, wanita 77th, sudah dirawat selama 5 hari di RS. Hasil
pemeriksaan klinis dan biokimiawi menunjukkan adanya perbaikan. Hasil pemeriksaan
status fungsionalnya belum menunjukkan adanya perbaikan. Kemungkinan yang bisa terjadi
adalah adanya:
A. Depresi
B. Demensia
C. Gagal pulih
D. Anemia
E. Semua benar
154.Batuk paru dan batuk hebat pada batuk kronis, Hemoptisis, Mengi/strindor, Gambaran
kavitas, Gambaran atelektasi, merupakan gejala tumor paru dari aspek:
A. Gejala ekstratorasik metastatik
B. Gejala intrapulmoner
C. Gejala ekstratorasik non metastatik
D. Gejala stadium awal tumor paru
E. Gejala intratorasik ekstrapulmonal
155.Seorang pasien berumur 55 tahun, masuk RS dengan cerebrovaskular attack, batuk, maupun
sesak. Setelah 5 hari kemudian pasien menjadi demam dan disertai batuk berdahak
kekuningan. Pada pemeriksaan fisik didapatkan suhu 39 C, pada foto rontgen didapatkan
infiltrat pada paru kanan. Dari hasil pemeriksaan darah didapatkan leukosit 26000/mm3.
Diagnosis yang mungkin:
A. Pneumonitis
B. Pneumonia nosokomial
C. Bronchitis kronik
D. Pneumonia masyarakat
E. Pneumonia interstitialis
156.Seorang perempuan 54th, datang dengan keluhan sesak napas yang semakin meningkat.
Riwayat merokok 30 pak/thn. Pemeriksaan foto rontgen thorax: hiperinflasi dada,
pemeriksaan spirometri FEV1/FVC<70%, tidak berubah setelah diberi bronkodilator.
Intervensi apakah yang mungkin diberikan untuk mempertahankan kelangsungan hidup?
A. Terapi jangka panjang dengan steroid
B. Long acting anti cholinergic inhaler
C. Terapi inhalasi dengan SABA/SAMA
D. Antibiotik profilaksis
E. Berhenti merokok

157.Seorang pria 24 tahun datang ke Poli penyakit dalam dengan keluhan utama sesak, disertai
batuk berdahak warna putih kental banyak. Terutama saat asap motor dan cuaca dingin.
Sebelumnya pasien mengalami keluhan serupa yang kambuh 2-3x dalam seminggu dan
kambuh pada malam hari 2-3x/bulan. Ayah pasien ada riwayat alergi telur. Pemeriksaan
fisik wheezing ekspiratoar bilateral. Kadar IgE meningkat. Diagnosa yang paling tepat?
A. asma bronkiale persisten sedang
B. asma bronkiale intermitten
C. status asmatikus
D. asma bronkiale persisten ringan
E. asma bronkiale persisten berat

158.Laki-laki 35 tahun datang dengan keluhan sesak napas sejak 3 jam yang lalu. Dari
pemeriksaan foto thorax PA didapatkan gambaran hiperlusen tanpa gambaran vaskuler di
hemithorax dextra. Apakah diagnosa pada kasus ini?
A. Pneumonia
B. Asma Bronkhial
C. Pneumothorax
D. Emfisema
E. Efusi pleura

159.Lelaki 25th, datang sesak napas berulang dan sejak 2 hari yang lalu memberat. Pasien
merokok 1 hari 1 bungkus. Pasien datang dalam keadaan mengantuk, diajak bicara tidak
nyambung. Pemeriksaan fisik KU lemah, somnolen, bibir sianosis. RR 32x/menit. N’
120x/menit. TD 120/85. Dada barel chest, perkusi hipersonor, suara dasar menurun,
wheezing ekspirasi positif, rbh positif. Pemeriksaan gold standard?
A. Analisa gas darah
B. Spirometri
C. USG thoraks
D. LED
E. Peak flowmetry

160.Seorang laki-laki berusia 32 tahun, datang ke puskesmas dengan keluhan batuk. Batuk
dirasakan sejak 1 bulan yl, disertai demam ringan dan sering berkeringat di malam hari.
Pada pemeriksaan fisik didapatkan TD 120/70 mmHg, HR 80 x/menit, RR 24 x/menit,
temperatur 37,5 C. Pada pemeriksaan dada kanan didapatkan suara napas bronchial, perkusi
redup, tactile fremitus meningkat, dan suara napas tambahan ronkhi, sedangkan paru kiri
dalam batas normal. Pemeriksaan tambahan apakah yang paling tepat dilakukan pada pasien
ini:
A. Darah lengkap
B. Foto thoraks posisi PA
C. Dahak; pengecatan Ziehl-Neelsen
D. Laju endap darah
E. Test protein purified derivate

161.Seorang perempuan datang dengan keluhan sesak napas sebelumnya pasien mengaku 2 hari
yang lalu demam, batuk sangat produktif. Dahak banyak sekali dan bebau busuk seperti
karat besi. Pada pemeriksaan fisik: TD: 120/80mmHg, N: 110x/mnt, RR: 28x/mnt, T: 39 C
auskultasi terdapat ronkhi basah pada basal kanan-kiri. Diagnosis?
A. Efusi pleura
B. Kanker paru
C. Bronchitis
D. Pleuritis
E. Bronkiektasis infeksi

162.Pasien terinfeksi HIV dengan CD4 di bawah 350, mengalami Ko-infeksi TB Paru, maka
pengobatan ARV dan pengobatan TB Paru dengan pengaturan sebagai berikut:
A. Obat ARV ditunda sampai CD4 meningkat di atas 350, sementara obat TB Paru bisa
langsung diberikan
B. Obat ARV langsung diberikan bersamaan dengan pengobatan TB Paru
C. Obat ARV diberikan setelah pengobatan TB Paru selesai kategori I selama 6 bulan
D. Obat ARV pilihan pertama adalah: Iamivudin, Zidovudin, Nevirapin
E. Obat ARV diberikan setelah pengobatan TB Paru berlangsung selama 2 minggu- 8
minggu
163.Seorang wanita usia 57 tahun dengan tubuh kurus, ketika menyapu di depan rumah tiba-tiba
jatuh terduduk karena terpeleset, kejadian sudah berlangsung kurang lebih 3 bulan yang
lalu. Sampai saat masih dikeluhkan terasa nyeri pada pinggang, di samping itu juga terdapat
riwayat asma kronik sejak muda sering minum obat yang berwarna hijau (prednison). Hasil
pemeriksaan fisik: KU lemah, jalan agak pincang, tensi: 130/80mmHg, nadi 72/menit,
respirasi rate: 20x/menit, BB: 50 kg, TB: 170 cm. Di bawah ini merupakan faktor risiko
osteoporosis:
A. Aktivitas fisik menyapu
B. Pincang
C. Kurus
D. Asma bronkiale persisten berat
E. Hipertensi
164.Seorang laki-laki berusia 68 tahun datang ke puskesmas dengan keluhan sesak napas bila
berganti baju. Keluhan sesak sering kambuh-kambuhan dalam 1 tahun ini. Pasien juga
mengeluh batuk berdahak warna putih, dan tidak disertai panas badan. Pasien adalah
seorang perokok berat. Pada pemeriksaan fisik didapatkan TD 120/70 mmHg, HR
100x/menit RR 30x/menit, temperature 37°C. Pada pemeriksaan didapatkan bentuk dada
barrel chest, stem fremitus normal, hipersonor pada dada kiri dan kanan serta didapatkan
bunyi wheezing dada kiri maupun kanan. Apakah diagnosis pasien tersebut?
A. Penyakit paru obstruksi kronis grade C-D
B. Asma bronkiale persister berat
C. Tuberkulosis paru
D. Empyma
E. Sindroma obstruksi pasca tuberculosis
165.Seorang laki-laki 35 tahun datang ke Poliklinik Penyakit Dalam untuk melakukan general
check up rutin tahunan. Pasien merupakan pekerja tambang batu bara sudah 5 tahun. Saat
ini pasien tidak mengeluh batuk, tidak mengeluh sesak, tidak ada bengkak pada tungkai.
JVP R+1, Pemeriksaan fisik didapatkan dalam batas normal. X foto thorax PA didapatkan
adanya nodul bulat, diameter 3x3 cm, di perifer pulmo dextra lobus superior. Dokter
menyarankan dilakukan biopsi transthorakal, tetapi pasien masih pikir-pikir. Satu tahun
kemudian pasien tersebut kembali datang ke poliklinik dengan keluhan nyeri dan bengkak
seluruh sendi-sendi jari tangan kiri dan kanan, pergelangan tangan kanan dan kiri, sudah
mulai dirasakan 8 minggu terakhir. Hasil laboratorium menunjukkan LED 32 mm/jam, CRP
27 mg/dl. Kemungkinan diagnosis pasien tersebut yang paling tepat adalah:
A. felty syndrome
B. Caplan syndrome
C. Silikosis
D. Berylliosis
E. Coal workers pneumoconiosis

166.Seorang laki-laki 36 tahun datang ke IGD dengan keluhan sesak nafas berat sejak 3 hari
sebelum masuk rumah sakit, sesak terus menerus, batuk kering, tidak ada demam. Pasien 3
tahun ini bekerja sebagai pemotong keramik. Sebelum bekerja di pabrik tersebut pasien
melakukan medical check up dan dikatakan kondisi baik oleh dokter. Hasil pemeriksaan
fisik didapatkan laju pernafasan 28x/menit, perkusi dada redup pada hemithorax superior
dextra dan sinistra. Dilakukan pemeriksaan X foto thorax PA didapatkan hasil nodul pada
lobus superior paru dextra et sinistra dengan kalsifikasi. Pewarnaan BTA dan KOH hasil
negatif. Pemeriksaan darah rutin dalam batas normal, ureum 68 gr/dl kreatinin 1,8 gr/dl
SGOT 88 U/L SGPT 98 U/L. Apakah diagnosis yang paling tepat untuk kondisi di atas :
A. Silikosis kronik
B. Silikosis akut
C. Silikosis cepat
D. Berylliosis akut
E. Asbestosis

167.Seorang laki-laki 28 tahun datang ke IGD dengan keluhan sesak nafas, awalnya dirasakan
jika melakukan olah raga, tetapi 1 bulan ini semakin memberat, selain itu juga dikeluhkan
batuk tidak berdahak, tidak ada keluhan terbangun malam hari karena sesak, kaki bengkak,
berat badan turun, diare lama, maupun demam lama. Pasien bekerja selama 2 tahun di
Pabrik alumunium. Pasien sudah periksa ke dokter dikatakan bronkitis. Pemeriksaan fisik
didapatkan mengi pada kedua basal paru, jantung dalam batas normal, tidak terdapat oedem
dan clubbing finger pada extremitas. Hasil X foto Thorax PA didapatkan nodul multipel
kecil-kecil dengan fibrosis pada kedua basal paru, dilakukan transthorakal biopsi dengan
guided CT didapatkan hasil radang kronis non spesifik dengan reaksi granulomatosa, tak
tampak tanda keganasan. Berdasarkan data tersebut, apa diagnosis yang paling tepat pada
pasien di atas:
A. Asbestosis
B. Berylliosis
C. Silikosis
D. Coal workers pneumoconiosis
E. Bagasosis
168.Wanita 78 tahun dibawa ke RS dengan sesak nafas berat sejak 2 hari yang lalu. Kondisi di
IGD menunjukkan saturasi oksigen SpO2 60% yang kemudian meningkat menjadi 82%
dengan pemberian oksigen masker non rebreathing 8-10 liter per menit selama 30 menit,
hasil foto thorax menunjukkan gambaran pneumonia multi lobaris. Pasien selanjutnya
diintubasi dan dipasang ventilator mekanik dengan assist-control mode, RR 24X/menit,
Volume Tidal 6mL/kg, FiO2 1.0, PEEP 12 cmH2O, lalu dilakukan BGA dengan hasil pH
7,20, PCO2 32 mmHg, PO2 54mmHg, A-aDo2: 200. Mekanisme terjadinya hipoksemia
pada kasus ini:
A. Hipoventilasi
B. Hipoventilasi dan V/Q mismatch
C. V/Q mismatch
D. Shunting
E. Gangguan difusi

169.Laki-laki 27 tahun telah didiagnosis menderita muscular dystrophy, datang ke poliklinik


untuk pemeriksaan kesehatan. Pasien tidak mengeluh sesak nafas, pada pemeriksaan
didapatkan saturasi oksigen (SpO2) 86% pada udara ruangan. Pemeriksaan jantung dan paru
normal. Foto thorax menunjukkan volume paru yang mengecil , lalu dilakukan BGA dengan
hasil pH 7,18, PCO2 64 mmHg, PO2: 78 mmHg, HCO3: 36, A-aDo2: 10. Pemeriksaan
spirometri menunjukan restriksi. Mekanisme terjadinya hiperkapnia pada kasus ini:
A. Hipoventilasi
B. V/Q mismatch
C. Gangguan perfusi
D. Shunting
E. Gangguan difusi
170.Seorang wanita 32 tahun, sedang hamil 36 minggu dikonsulkan dari bagian obsgyn dengan
sesak nafas mendadak, tidak demam,. Selama kehamilan tidak pernah mengalami masalah
medis, tidak ada obat yang rutin diminum selain vitamin kehamilan. Pada pemeriksaan
didapatkan TD 128/78 mmHg, HR 126 x/menit, RR 28x/menit, saturasi oksigen (SpO 2)
96%, jantung dan paru dalam batas normal, terdapat pitting oedeme pada kedua kaki. Hasil
X foto thorax PA menunjukkan hasil normal, EKG didapatkan S pada Lead I, Q patologis
dan t inverted pada lead III. Pada BGA didapatkan pH 7,28, PaCO 2 50 mmHg, PaO2 85
mmHg, A-aDO2: 150. Pemeriksaan lanjutkan yang paling tepat adalah:
A. Skintigrafi
B. Ekokardiografi
C. CT Angiografi paru
D. USG Dopler kaki
E. CT scan thorax

171.Seorang wanita 30 tahun datang ke poli penyakit dalam dengan keluhan sering sesak nafas,
dada kadang terasa nyeri, sesak memberat saat beraktivitas. Tanda vital dalam batas normal,
tidak didapatkan whezing maupun rongki pada auskultasi paru. Tidak didapatkan murmur.
Spirometri tidak didapatkan kelainan, echokardiografi didapatkan kesan dilatasi pada
ventrikel kanan, atrium kiri dan ventrikel kiri kesan normal. TAPSE 18 mm, LVEF 70%
dan RVSP 55 mmHg. Kemudian pasien dilakukan pemeriksaan katerisasi jantung
didapatkan Pulmonary artery (PA) systolic pressure 48 mmHg, PA diastolic pressure 20
mmHg, PA mean pressure 34 mmHg, respon terhadap vasodilatasi PA mean pressure turun
12 mmHg. Diagnosis yang tepat pada kasus ini adalah
A. Kor Pulmonal Kronik
B. Emboli paru kronik
C. ASD
D. Hipertensi pulmonal primer
E. Hipertensi pulmonal sekunder
172.Seorang wanita 28 tahun datang ke poli penyakit dalam dengan keluhan sering sesak nafas,
dada kadang terasa nyeri, sesak memberat saat beraktivitas. Pada spirometri didapatkan
kesan restriksi ringan, dengan echokardiografi kesan dilatasi pada ventrikel kanan. Atrium
kiri dan ventrikel kiri kesan normal. TAPSE 20 mm, LVEF 70% dan RVSP 55 mmHg.
Kemudian pasien dilakukan pemeriksaan katerisasi jantung didapatkan Pulmonary artery
(PA) systolic pressure 50 mmHg, PA diastolic pressure 30 mmHg, respon terhadap
vasodilatasi MPAP turun 15 mmHg. Terapi yang tepat pada kasus ini adalah
A. Inhalasi iloprost 200 mcg/4 jam
B. Sildenafil 50 mg/ hari
C. Nifedipine 30 mg/hari
D. Diltiazem 540 mg/hari
E. Riociquat 1,5 mg/8 jam

173.Seorang laki-laki 70 tahun dating ke poliklinik keluhan sering sesak nafas, mengi, riwayat
merokok 20 batang per hari selama 40 tahun. Pemeriksaan fisik didapatkan JVP R + 0,
suara eksperium diperpanjang pada kedua lapang paru, jantung dalam batas normal, dan
tidak didapatkan edema ektremitas. Pada pemerikssan X Thorax PA dengan gambaran
hiperinflasi, sela iga melebar dan Palla’s sign. Komplikasi PPOK yang mulai muncul pada
kasus ini adalah
A. Gagal nafas tipe II
B. Hipertensi pulmonal
C. CPC
D. Pneumothorax
E. Chronic thromboemboli pulmonary hypertension

174.Seorang laki-laki 75 th datang ke IGD karena keluhan sesak nafas progresif sejak 4 tahun
terakhir. Pasien ada riwayat merokok 12 batang per hari. Pasien pernah dirawat di rumah
sakit karena keluhan yang sama 1 bulan terakhir. Dilakukan penilaian mMrc 3 dan CAT 12 .
Pada pemeriksaan fisik didapati tampak sesak nafas, purse lift breathing, thorak
emfisematous, retraksi interkostal, tampak clubbing finger. Tekanan darah 120/80 mmHg,
HR: 110x/mnt, RR: 24x/menit dengan saturasi oksigen (SpO2) 88%. Pada pemeriksaan
paru ditemukan ekspirasi memanjang. Pemeriksaan laboratorium didapatkan Hb 13,2 g/dl,
hematokrit 51%, lekosit 6000, trombosit 320.000. Analisis gas darah pH 7,3, PaO2 60,
PCO2 65, HCO3 34. Pasien sudah sering menggunakan obat inhalasi kortikosteroid, LAMA
dan LABA namun masih sesak nafas. Maka Pemberian terapi oksigen jangka panjang di
rumah pada pasien ini yang tepat adalah:
A. Pemberian oksigen 1-2 lpm dengan nasal kanul
B. Pemberian oksigen 3-4lpm dengan nasal kanul
C. emberian oksigen 4 lpm dengan masker non rebreathing
D. Pemberian oksigen 6-8 lpm dengan masker rebreathing
E. Pemberian oksigen 6-8lpm dengan masker venture

175.Seorang wanita 67 th datang periksa ke poliklinik Penyakit Dalam karena keluhan sesak
nafas sejak 3 tahun terakhir. Pasien sudah sering menggunakan obat inhalasi LABA, LAMA
dan obat salbutamol untuk mengurangi sesak nafasnya. Pada pemeriksaan fisik didapatkan
tampak sesak nafas, purse lift breathing, thorak emfisematous, retraksi interkostal, sternal
lift,dan tak tampak clubbing finger dan oedem. Tekanan darah 100/80 mmHg, HR:
110x/mnt, RR:27x/menit dengan saturasi oksigen 90%. Pada pemeriksaan paru ditemukan
ekspirasi memanjang. Pemeriksaan laboratorium didapatkan Hb 13,2 g/dl, hematokrit 39%,
lekosit 5800/mm3, trombosit 280.000/mm3. Manakah pernyataan yang benar mengenai
indikasi pemberian oksigen jangka panjang pada pasien diatas adalah:
A. Indikasi pemberian oksigen kontinyu apabila PaO2 selama latihan turun ≤ 55mmHg
B. Indikasi pemberian oksigen kontinyu apabila PaO2 saat tidur turun ≤ 55mmHg dengan
komplikasi hipertensi pulmoner, somnolen dan aritmia
C. Indikasi pemberian oksigen kontinyu apabila saat istirahat PaO2 turun 56-59 mmHg
D. Indikasi pemberian oksigen kontinyu apabila selama latihan saturasi turun ≤ 88%
E. Indikasi pemberian oksigen kontinyu bila kondisi hematokrit > 50%

176.Seorang laki-laki 68 th datang ke poliklinik Penyakit Dalam karena keluhan sesak nafas
sejak 5 tahun terakhir. Sebulan yang lalu penderita mondok di rumah sakit dengan keluhan
yang sama, riwayat merokok sejak 40 tahun. Pada pemeriksaan fisik didapatkan tampak
sesak nafas, purse lift breathing, thorak emfisematous, retraksi interkostal, sternal lift,dan
tampak clubbing finger. Tekanan darah 110/80 mmHg, HR: 110x/mnt, RR:28x/menit
dengan saturasi oksigen 90%. Pada pemeriksaan paru ditemukan ekspirasi memanjang.
Pemeriksaan laboratorium didapatkan Hb 14,7 g/dl, hematokrit 39%, lekosit 5800/mm 3,
trombosit 280.000/mm3. Manakah pernyataan yang benar mengenai indikasi pemberian
oksigen tidak kontinyu pada pasien diatas adalah:
A. apabila PaO2 selama istirahat turun ≤ 55mmHg
B. apabila selama istirahat saturasi SpO2 ≤ 88%
C. apabila saat latihan PaO2 ≤ 55 mmHg
D. apabila selama latihan saturasi > 88%
E. apabila kondisi hematokrit > 50%

177.Seorang laki-laki 42 tahun datang ke Poli Penyakit Dalam untuk kontrol rutin. Pasien
memiliki riwayat DM 10 tahun ini, terkontrol. Akhir-akhir ini pasien mengeluh sering
mengantuk dan sulit berkonsentrasi saat bekerja. Istrinya mengatakan bahwa pasien sering
mengorok dan kadang mengeluarkan suara seperti tercekik saat tidur. Pemeriksaan fisik
didapatkan BMI 28 kg/m2. GDP 93 mg/dl, GD2PP 120 mg/dl, HbA1c 6,5%.Pemeriksaan
penunjang yang menjadi gold standard untuk menegakkan diagnosis kasus diatas adalah:
A. Elektrokardiografi
B. Elektroensefalografi
C. Spirometri
D. Polisomnografi
E. X-foto thorax

178.Seorang wanita 25 tahun dating ke Poli Penyakit Dalam dengan keluhan sering mengantuk
dan mudah lelah. Pasien sering mengorok dan kadang mengeluarkan suara seperti tercekik
saat tidur. Pasien telah menjalani pemeriksaan penunjang dan didiagnosis menderita
obstructive sleep apnea. Skala yang dapat digunakan untuk mengetahui kuantitas dan
derajat gangguan tidur pada pasien sleep apnea adalah:

A. Skala Epworth
B. Skala Morse
C. Skala Borg
D. Skala FAST
E. Skala Ranson

179.Seorang laki-laki usia 48 tahun kontrol ke Poli Penyakit Dalam karena menderita
obstructive sleep apnea. Pasien mengorok saat tidur dan beberapa kali terbangun di malam
hari. Pada pemeriksaan didapatkan BMI 30 dan indeks gangguan respirasi 7. Pemeriksaan
paru dalam batas normal. Pada kunjungan sebelumnya, pasien sudah disarankan
menggunakan CPAP, namun ia merasa kesulitan dan tidak dapat menggunakan alat
tersebut. Pilihan terapi alternatif pada pasien ini adalah:

A. Trakeostomi
B. Penggunaan alat penopang mulut
C. Reseksi parsial lidah
D. Uvulopalatofaringoplasti
E. Penggunaan oksigen dirumah

180.Seorang laki-laki 42 tahun dikonsulkan ke penyakit dalam, karena sesak nafas. Dua hari
yang lalu pasien menjalani operasi patah tulang femur. Dari pemeriksaan didapatkan tanda
vital Tensi 140/80 mmHg, HR 106x/mnt, RR 30x/mnt. Pemeriksaan Foto Thoraks tidak
didapatkan pembesaran jantung, westermark sign dan hamptom sign. Pemeriksaan EKG
gelombang S di lead I, gelombang Q dan T di lead III. Diagnosis pasien tersebut adalah:

A. Pneumonia
B. Emboli paru
C. Aspirasi
D. Atelektasis
E. Edema Paru

181.Seorang laki-laki 60 tahun datang ke IGD dengan keluhan nyeri dada 1 hari, tidak
menghilang dengan istirahat, nyeri dada dirasakan saat menarik nafas. Anamnesis tidak
didapatkan riwayat DM, jantung, paru namun terdapat pengobatan prostat stadium IV dan
pasien sehari – hari hanya berbaring. Dari pemeriksaan didapatkan tanda vital Tensi
150/100 mmHg, HR 110x/mnt, RR 30x/mnt SpO2 88%. Pemeriksaan EKG gelombang S di
lead I, gelombang Q patologis dan T di lead III. Pemeriksaan Foto Thoraks terdapat
pembesaran arteri pulmonalis kanan desenden. Gambaran radiologi yang terdapat pada
pasien tersebut dikenal sebagai
A. Tanda Wasternmark
B. Tanda Fleischner
C. Tanda Hampton
D. Tanda Palla
E. Tanda Polo Mint

182.Seorang wanita 40 tahun datang ke IGD karena sesak nafas. Aloanamnesis dengan keluarga
pasien didapatkan bahwa pasien sudah 3 tahun pasien menderita kebanyakan sel darah
merah dan setahun ini sudah dilakukan tindakan flebotomi sebanyak 2 kali. Dari
pemeriksaan didapatkan pasien gelisah tanda vital Tensi 80/60 mmHg, HR 106x/mnt
lemah, RR 30x/mnt, SpO2 88% . Pasien sudah mendapat cairan yang cukup dan telah
terpasang vasontriktor dengan dosis optimal. Pemeriksaan radiologi thorak didapatkan
westermark sign. Pada EKG gelombang S di lead I, gelombang Q patologis dan T di lead
III. Saat diberikan bolus heparin 3000 Unit pasien mengalami gatal berbentol,sehingga
pemberian antikoagulan dihentikan. Tindakan untuk keluhan utama sesak nafas selanjutnya
yang dilakukan adalah
A. Aspilet
B. clopidogrel
C. Ticagleror
D. Streptokinase
E. Asam traneksamat

183.Seorang laki-laki usia 34 tahun datang ke UGD dengan keluhan sesak nafas setelah terkena
debu saat pasien membersihkan karpet di kamar. Keluhan disertai hidung gatal, bersin-
bersin, batuk berdahak warna jernih encer, mengi. Pasien memiliki riwayat asma dan
riwayat alergi sebelumnya. Pasien juga merokok 2 bungkus perhari sejak 14 tahun lalu.
Riwayat asma pada keluarga tidak diketahui. Pemeriksaan fisik didapatkan kesadaran
compos mentis, frekuensi nafas 28x/menit, tekanan darah 120/70, suhu 37 oC, nadi
92x/menit. SaO2 93%. Pemeriksaan fisik tidak ditemukan barrel chest, terdapat wheezing
pada seluruh lapangan paru. Pasien sudah dinebulisasi salmeterol, budesonide, tiotropium.
Pasien juga mendapat montelukast per oral, namun keluhan sesak masih dirasakan. Hasil
laboratorium Hb 15gr/dL, leukosit 12000/mm3. hitung jenis didapatkan eosinofil 60%,
neutrofil 10%. Tinggi badan 170cm, berat badan 70kg. Pasien membawa hasil pemeriksaan
spirometri sebagai berikut :
Normal Observed % Prediksi Post bronkodilator
FVC (L) 3,52 3.32 95
FEV1 (L) 3,17 2,06 65 2,21
FEV1/FVC (%) 80 54,4 68

Tatalaksana medikamentosa selanjutnya yang dapat dipertimbangkan pada pasien tersebut


adalah :
A. azithromycin
B. aminofilin
C. roflumilast
D. erythromycin
E. omalizumab

184.Seorang perempuan usia 24 tahun datang ke poliklinik dengan keluhan batuk yang sudah
dirasakan 1 bulan. Batuk dikeluhkan saat malam hari. Batuk tidak berdahak. Tidak ada
sesak nafas, demam, penurunan berat badan, keringat malam hari. Riwayat asma
sebelumnya disangkal, riwayat alergi makanan disangkal. Riwayat asma pada keluarga
disangkal. Pemeriksaan fisik didapatkan kesadaran compos mentis, frekuensi nafas
20x/menit, tekanan darah 120/70, suhu 37oC, nadi 92x/menit. SaO2 98%. Pemeriksaan fisik
tidak didapatkan barrel chest dan suara tambahan paru. Didapatkan hasil pemeriksaan
spirometri sebagai berikut :
Normal Observed % Prediksi
FVC (L) 3,52 3.32 95
FEV1 (L) 3,17 3 94
FEV1/FVC (%) 80 90 98

Pemeriksaan penunjang yang diperlukan pada pasien ini adalah


A. X foto thorax
B. sputum BTA
C. tes metakolin
D. skin prick test
E. tes tuberkulin

185.Seorang laki-laki berusia 29 tahun datang ke Poliklinik Penyakit Dalam dengan keluhan
batuk berdahak sejak 10 tahun terakhir dan dirasakan semakin lama semakin memberat.
Pasien juga mengeluhkan sesak nafas dan batuk terkadang bercampur darah. Nafsu makan
menurun, berat badan menurun dan terkadang merasakan demam. Pasien sudah menikah
selama 6 tahun dan belum memiliki anak. Riwayat ibu pasien meninggal dunia pada usia
muda akibat penyakit paru. Pada pemeriksaan fisik didapatkan barrel chest, suara dasar
paru bronkial dan ronki pada bagian apeks paru. Pada foto thoraks PA tampak gambaran
kistik pada lobus atas paru. Pemeriksaan gold standard diagnostik yang diperlukan pada
pasien ini, adalah:

A. Spirometri
B. Pemeriksaan TCM sputum
C. Genotyping darah
D. Uji Klorida Keringat
E. Bronchoalveolar Lavage (BAL)

186.Seorang wanita berusia 30 tahun datang ke Poliklinik dengan keluhan batuk berdahak yang
dirasakan semakin memberat sejak 7 hari terakhir. Pasien sebelumnya sering mengalami
keluhan yang sama sejak berusia 16 tahun tetapi tidak melakukan pengobatan rutin ke
dokter. Saat ini batuk semakin memberat, kadang-kadang bercampur darah disertai sesak
nafas dan terdapat gangguan pada pencernaan. Pada pemeriksaan fisik BMI 16 kg/m 2, bntuk
thoraks barrel chest, penggunaan otot-otot bantu pernafasan, ronki basah kasar apeks paru
paru kanan dan clubbing finger. Foto thoraks PA tampak gambaran infiltrat pada lobus
apeks, foto sinus tampak gambaran parasinusitis dan uji klorida keringat 87 mmol/L.
Berikut pernyataan yang benar mengenai kasus di atas adalah

A. Kelainan genetik yang bersifat resesif heterogen dengan gambaran patobiologik yang
mencerminkan mutasi pada gen Cystic Fibrosis Transmembrane Conductance
Regulator
B. Kelainan genetik yang bersifat dominan dengan gambaran patobiologik yang
mencerminkan mutasi pada gen Cystic Fibrosis Transmembrane Conductance
Regulator
C. Rata-rata pasien dapat hidup lebih dari 52 tahun untuk perempuan dan 49 tahun untuk
laki-laki
D. Masalah paru lain yang dapat dijumpai Allergic Bronchopulmonary Aspergilus
(ABPA), Abses Paru dan Tuberkulosis
E. Penyakit autosomal dominan akibat mutasi gen yang terletak pada kromosom 7
187.Seorang wanita berusia 32 tahun datang ke Rumah Sakit dengan keluhan batuk berdahak
yang dirasakan semakin memberat sejak 10 hari terakhir. Pasien sebelumnya sering
mengalami keluhan yang sama sejak berusia 14 tahun tetapi tidak melakukan pengobatan
rutin ke dokter. Saat ini batuk semakin memberat, sputum yang purulen kadang-kadang
bercampur darah disertai sesak nafas dan gangguan pencernaan. Pada pemeriksaan fisik
didapatkan keadaan umum pasien tampak kurus, barrel chest, penggunaan otot-otot bantu
pernafasan, ronki paru pada kedua apeks paru dan clubbing finger. Foto thoraks PA tampak
gambaran infiltrat pada lobus apeks, foto sinus tampak gambaran parasinusitis dan uji
klorida keringat 80 mmol/L. Kriteria diagnosis yang baku untuk menegakkan diagnosis
pada pasien ini adalah:

A. Uji klorida keringat, dan gambaran klinis yang khas tipe gastrointestinal serta
azoospermia obstruktif
B. Uji klorida keringat, gambaran klinis yang khas tipe paru, serta membutuhkan
penunjang uji faal paru
C. Uji klorida keringat, dan gambaran klinis yang khas tipe paru atau tipe gastrointestinal
D. Uji klorida keringat, gambaran yang khas tipe gastrointestinal, serta membutuhkan
penunjang Genotyping
E. Uji klorida keringat, gambaran klinis yang khas tipe gastrointestinal atau tipe paru,
serta membutuhkan penunjang analisa semen dan BAL

188.Laki laki 56 tahun datang ke UGD dengan keluhan sesak napas. Pasien mengeluh juga ada
nyeri dada. Riwayat pasien pernah di rawat di RS daerah sebelumnya dengan bengkak kaki
kanan. Pasien terbaring di tempat tidur kurang lebih 2 bulan ini. Pada pemeriksaan tanda
vital didapatkan Tekanan darah 80/70 mmhg, nadi 110 kali/menit, RR 32 kali/menit.
Pemeriksaan fisik didapatkan JVP meningkat, hepatojugular refleks meningkat. Pasien
membawa hasil laborat Hb 11,2 gr/dl, Leukosit 4500/Ul, D dimer 2920 mg/dl. EKG
didapatkan S di Lead I, Q patologis di lead III dan T inverted di lead III. USG Doppler
tampak adanya DVT pada vena femoralis dextra. Diagnosis yang paling mungkin pada
pasien ini adalah :

A. Emboli Paru
B. Gagal jantung kiri
C. Diseksi aorta
D. Pneumonia aspirasi
E. Sindrom koroner akut

189.Seorang wanita 37 tahun datang ke UGD dengan keluhan sesak napas terutama jika
aktifitas, cepat lelah dan nyeri dada. Pasien juga mengeluhkan kadang pingsan. Pada
pemeriksaan fisik di dapatkan adanya Fenomena Raynaud’s. Pasien membawa foto thorak
dengan hasil adanya pembesaran hilus dan pembesaran ventrikel kanan. Pada
elektrokardiografi didapatkan adanya deviasi aksis ke kanan, hipertrofi ventrikel kanan dan
strain ventrikel kanan. Penyebab hipertensi pulmonal yang paling mungkin pada kasus ini
adalah :

A. Connective tissue disease


B. Hipertensi pulmonal idiopatik
C. PPOK
D. Emboli paru kronik
E. Atrial septal defect

190.Laki laki 60 tahun datang ke UGD dengan keluhan sesak napas dan nyeri dada. Riwayat
pasien pernah di rawat di RS daerah sebelumnya dengan bengkak kaki kanan. Pasien
terbaring di tempat tidur kurang lebih 2 bulan ini. Pada pemeriksaan tanda vital didapatkan
Tekanan darah 75/65 mmhg, nadi 125 kali/menit, RR 40 kali/menit. Pemeriksaan fisik
didapatkan JVP R+4 cm, hepatojugular refleks (+), kedua kaki tidak bengkak. USG Doppler
tampak adanya deep vein thrombosis di vena femoralis dekstra. Tatalaksana awal yang bisa
diberikan untuk mengatasi kegawatannya adalah:

A. Pemberian antikoagulan dengan heparin


B. Tindakan pembedahan embolektomi paru
C. Pemberian antikoagulan warfarin
D. Pemberian cairan infus untuk mempertahankan kestabilan keluaran ventrikel kanan dan
aliran darah pulmonal
E. Pemberian streptokinase 250.000 unit

191.Seorang wanita 32 tahun hamil 16 minggu dengan Ko infeksi TB-HIV dalam pengobatan
OAT kategori 1. Pada akhir pengobatan bulan ke-3, masih didapatkan keluhan sesak nafas
dan batuk berdahak dan dilakukan pemeriksaan BTA didapatkan hasil 2 positif. Pada
pemeriksaan GeneXpert saat itu didapatkan hasil M.Tuberkulosis detected rifampisin
resistant. Berdasarkan Panduan Tatalaksana TB MDR WHO 2016 pilihan regimen terapi
untuk pasien tersebut adalah
A. Klaritromisin dan makrolida lainnya direkomendasikan untuk pengobatan TB-MDR
pasien ini
B. Shorter regimen MDR-TB yang terdiri dari fase intensif 4-6 bulan dan 5 bulan fase
lanjutan
C. Regimen individual Tb MDR yang terdiri dari 8 bulan fase intensif dan 12 bulan fase
lanjutan tanpa OAT lini ke 2 golongan injeksi dan ethionamide.
D. Regimen Ofloxacin merupakan salah satu fluoroquinolone yang bisa dipilih pada
wanita hamil
E. Bedaquilline dan Delamanid tidak bisa digunakan karena memiliki efek samping
pemanjangan interval QT

192.Seorang laki-laki berusia 36 tahun,sejak 3 tahun lalu diketahui terinfeksi HIV dan
mendapatkan terapi antiretroviral. Berat badan menurun 10 kg selama 2 bulan terakhir dan
terdapat penurunan progresif jumlah limfosit CD4 sampai nilai 94/mm2. Dua minggu
sebelum berobat ke IGD, pasien mengeluh sesak nafas, terdapat batuk namun tidak
produktif. Pada pemeriksaan fisik pasien tampak sesak, tekanan darah 130/80 mmHg,
frekuensi nadi 112x/menit, RR 30x/menit, suhu 37.9 C. Pada auskultasi paru didapatkan
suara nafas bronkovesikuler. Pemeriksaan fisik abdomen tidak didapatkan organomegali.
Dari pemeriksaan darah rutin didapatkan hasil Hb 9,2, lekosit 11.500, trombosit 250.000
dan LED 68mm/jam. Pada foto rontgen thorax didapatkan gambaran infiltrate interstitial
difus luas. Berdasarkan data diatas, kemungkinan organisme penyebab infeksi oportunistik
pada pasien:
A. Cytomegalovirus
B. Cryptosporidia sp
C. Pneumocystis jiroveci
D. Cryptococcus Neoformans
E. Mycobacterium Avium Complex

193.Seorang laki laki 50 tahun datang ke poli penyakit dalam dengan keluhan sesak nafas yang
timbul mendadak setelah pasien bekerja di pengeringan gandum. Keluhan sesak disertai
batuk, demam ngelemeng, lemas, sakit kepala. Pasien mengatakan tidak merokok dan
tidak pernah sakit paru paru. Dari pemeriksaan fisik didapatkan TD : 120/70 mmHg, HR:
110x, rr: 24x t: 38 C thorak: terdapat ronkhi dikedua paru. Rontgen thorak : konsolidasi
difus. Laboraturium leukositosis dan neutrofilia. Terapi yang tepat untuk mengatasi
serangan akut tersebut adalah :
A. Ceftriakson
B. Azitromisin
C. Levofloksasin
D. Flukonazole
E. Kortikosteroid

194.Seorang laki laki usia 55 tahun datang ke poli penyakit dalam dengan keluhan mendadak
sesak nafas setelah melakukan pengeringan gandum. Sesak disertai batuk, demam, dan
lemas. Pemeriksaan fisik didapatkan TD : 120/70 mmHg, HR: 110x, rr: 24x t: 38 C
thorak: terdapat ronkhi dikedua paru. Rontgen thorak : konsolidasi difus. Laboraturium
leukositosis dan neutrofilia Setelah diperiksa pasien dinyatakan mengalami pneumonitis
hipersensitif. Apakah organisme penyebab dari pneumonitis hipersensitif ini?
A. Thermophilic actinomycetes
B. Aspergilosis fumigatus
C. Cryptokokus neoforman
D. Histoplasma capsulatum
E. Nocardia sp

195.Seorang perempuan 29 tahun datang ke Poliklinik penyakit dalam dengan keluhan sesak
nafas. Sesak nafas dirasakan saat aktivitas dan disertai batuk kering tidak berdahak.
Keluhan ini sudah dirasakan selama 1 tahun. Pasien sudah berobat ke dokter. Suami pasien
bekerja di pabrik pembuat atap rumah berbahan asbes sudah 2 tahun. Pasien merupakan ibu
rumah tangga, tugas mencuci baju suami dan anak-anak ia lakukan sendiri. Pemeriksaan
fisik didapatkan redup kedua basal paru. Hasil x foto thorax di dapatkan garis-garis opasitas
kecil di basis paru sampai pleura. Hasil spirometri di dapatkan kelainan faal paru didapatkan
penurunan FVC, FEV1, FEV1/ FVC turun. Komplikasi jangka panjang yang paling mungkin
dialami pasien tersebut diatas adalah
A. Atelektasis
B. Malignant mesothelioma
C. Pneumothorak
D. Penyakit Paru intersitial
E. Abses paru

196.Laki-laki 53 tahun masuk ke IGD dengan keluhan demam mendadak, menggigil,


berkeringat, lemas, dan sesak nafas dengan nafas pendek-pendek. Pasien tidak ada riwayat
asma sebelumnya. Pasien adalah seorang petani tembakau yang baru saja mendapat proyek
mengerjakan sawah milik orang lain yang sedang panen. Pada pemeriksaan fisik pasien TD
130/80 HR 110 RR 28 SpO2 92%. Pemeriksaan paru didapatkan ronki basah pada kedua
basal paru, tachipneu, tidak ada wheezing. Ro thorax didapatkan infiltrate pada kedua lobus.
Pada pemeriksaan faal paru pasien ini akan didapatkan:

A. Normal
B. Penurunan FVC dan FEV1 , FEV1/FVC menurun
C. Peningkatan FVC dan FEV1, FEV1/FVC meningkat
D. Penurunan FVC dan FEV1 , FEV1/FVC meningkat
E. Peningkatan FVC dan FEV1, FEV1/FVC menurun
197.Seorang wanita 55 tahun datang ke UGD dengan keluhan sesak nafas dan juga mengeluh
mudah lelah. Penderita mempunyai riwayat sakit artritis rematoid dan selama 2 tahun ini
mendapatkan terapi Methotrexate. Pada pemeriksaan fisik didapatkan tekanan darah 110/70
mmHg, Nadi 90 x/ menit, pernafasan 26 x/menit, tidak demam, didapatkan distensi vena
leher, jantung didapatkan P2 mengeras, sternal lift (+), shifting dullness (+), dan edema
tungkai bawah. Dari X-foto thorak didapatkan pelebaran hilus dan kardiomegali, EKG
menunjukkan RAD dan P pulmonal di lead II. Penyakit paru yang dapat menyebabkan
kelainan jantung pada pasien ini adalah :
A. Penyakit paru obstruktif kronik
B. Interstitial lung disease
C. Obstructive sleep apnea
D. Bronkiektasis
E. Asthma COPD Overlap Syndrome

198.seorang laki-laki 50 tahun datang ke UGD dengan keluhan sesak nafas. Sesak nafas sudah
sering dirasakan sejak 3 tahun ini. Sesak nafas dirasakan bertambah sering 5 bulan terakhir
terutama saat aktivitas, dan merasa sering cepat lelah. Menurut keterangan istrinya, dia tidur
mengorok, mudah mengantuk saat siang hari. Pasien juga merokok sejak usia muda. Pada
pemeriksaan didapatkan dispnea, kesadaran kompos mentis. Tekanan darah 140/80 mmHg,
Nadi 100 x/menit, pernafasan 24 x/menit, berat badan 92 kg, tinggi badan 168 cm,
didapatkan distensi vena leher. Pada jantung didapatkan P2 mengeras, didapatkan pula
hepatomegali dan edema tungkai. Dari foto thorak didapatkan pelebaran hilus dan
pembesaran jantung ke kanan, EKG menunjukkan axis bergeser ke kanan dan P pulmonal di
lead II. Salah satu komplikasi yang dapat terjadi pada pasien ini adalah :
A. Gagal jantung kiri
B. Cor pulmonale
C. Penyakit Jantung Hipertensi
D. Stroke hemoragik
E. Penyakit paru obstruktif menahun
199.Seorang pasien laki-laki usia 47 tahun datang ke rumah sakit dengan keluhan wajah
kemerahan dan nyeri, pada pemeriksaan fisik didapatkan tekanan darah 160/100 mmHg,
Nadi: 100x/mnt, pernafasan: 18x/mnt, suhu: 37oC, pada mata didapatkan enoftalmus,
terdapat anhidrosis serta paraplegia. Pada pemeriksaan penunjang urin didapatkan Vinil
Mandelic Acid (VMA) serta pada CT scan thoraks didapatkan massa di mediastinum
posterior. Kemungkinan diagnosis adalah:
A. Germ cell neoplasma
B. Limfoma
C. Kista bronkogenik
D. Timoma
E. Ganglioneuroma
200.Seorang lelaki berusia 40 tahun, datang berobat ke poliklinik dengan riwayat batuk kronik
sejak 1 bulan sebelum berobat. Batuk dengan dahak yang biasanya berwarna kuning dan
kadang-kadang kehijauan. Tidak ada riwayat mengi, asma, gagal jantung kongestif, dan
penyakit refluks gastroesofageal. Pasien merokok selama 15 tahun sebanyak 1
bungkus/hari. Pada pemeriksaan paru didapati ronki basah kasar nyaring pada basal paru
kanan. Hasil rontgen torak didapatkan gambaran kista kecil-kecil pada paru kanan bawah
disertai bercak-bercak infiltrat disekitarnya. Hasil spirometri: FEV 1 80%; FVC 88%.
Patofisiologi yang mendasari kondisi tersebut adalah:
A. Keradangan parenkim paru
B. Obstruksi saluran napas kecil-sedang
C. Ketidakseimbangan ventilasi dan perfusi
D. Reaksi hipersensitivitas pada saluran napas
E. Dilatasi dan destruksi dinding bronkus

Anda mungkin juga menyukai